You are on page 1of 74

09.

06
ScoreTop:VIP Only Section:09.06

Topic: September Math (Updated on 09/27/06)


Forum Jump
1).d=c+8
2).d=1.2c

Discussion Link

Discussion Link2

163. Is X^3>y^2
1). y=x^2
2). x>x^2

Discussion Link

164. Two people are working on a equation x^2+bx+c=0, the first people
get two results x1=... and x2=..., but the numbers are incorrect, they are
the result for x^2+bx+d=0; the other people get two results x1=... and
x2=...,but the numbers are incorrect, they are the result for
x^2+ex+c=0. What is the correct solution of equation x^2+bx+c=0?

Discussion Link

Edited by GMATer on 20 September 2006 at 9:00pm

Back to Top

soft rock
GMAT Tutor Posted: 24 September 2006 at 10:22am | IP Logged
165. Given 452*37*16, add 1 to which one of the 7 digits will increase
the value of the product by less than 1000?

Joined: 20 October 2004 Discussion Link


British Virgin Islands
Posts: 1125 166. Is x greater than 13/11?
Gender: Not Specified 1). X>1.182
2). X<1.19

Discussion Link

167. y is not zero, is (2^x/y)^x<1?


1). x>y
2). y< 0

Discussion Link

168. X+Y = 1; X^2 + Y^2 = 3, XY=?

Discussion Link

169. A certain number of products A were sold out and the total cost was
5 percent of the sales value plus $100,000. If the sales made a profit, is
the number the products greater than 21,000?
1). The total sales value is $110,000
2). The price of the product is $5
Answer:
5X > 100,000 + 0.05*5X => X > 100,000/4.75 > 21,000, answer is b.

Discussion Link
08.05
ScoreTop:VIP Only Section:08.05

Topic: August Math Answers--Explanations(31/08)


Forum Jump
12.

95.If x and y are positive integers, (x+2)/(y+3)>(y+2)/(x+3)?


1) x*y=2
2) x>y
From I, we only get a quadratic equation and we cannot solve that.
From II, yes we can calculate. Plugin!!

Answer: B

96.The range of the male in a team is 14 inches, and the range of the
female in the team is 12 inches, what is the range of the team?
1) The shortest of the man is three inches more than the tallest woman.
2) The tallest man is 75 inches

From I, .. the sequnce is.. all the girls.. and then all the boys. Hence, the
range adds up, which is 28.
From II, No it is not possible.
Hence, A.
Answer: A

97.What is the average of x, y, and z?


1) The average of x and y is 4
2) The average of y and z is 6 & amp; amp; amp; amp; amp; amp;
amp; nbsp; ; ; ; ; ; ; ; ; ; ; ;

From I, it is not possible to determine the avg of all the three numbers. Z
is missin.
From II, same as I.
From I and II, not possible, since even the addition does not help.

Answer: E

98.The line k pass the two points (-4,-2) and (4,6) ,which line is
perpendicular to line k and intersect in the point (4,6)?
a, x-y-10=0
b, x+y-10=0
c, x+y-2=0

Firstly, if a line intersects at one of the point above, the point should
satisfy the eqn. Only B does that.
Also, the slope is -1 for the perpendicular line.
Answer: B

99.15% of all trees planted in the F and not flowered.40 percent of trees
planted in the F are flowered. What is the percent of trees planted in the
F?
a, 55% b,25%
Answer: B
all trees=t, In F tree=f,and then not flowered in F=0.15t,flowered in
F=0.4f ¨¤ 0.15t=0.06f, 0.06f:0.4f=0.15t:x, x=0.1, 0.1+0.15=0.25=25%
I think what it's trying to say is: out of all trees planted WHICH DID NOT
FLOWERED, trees in F which did not flower count as 15% OF THOSE; and
40% of F trees flowered, so what % is the number of trees planted in F
overall?
So 15% / 60% = 25%

100.On a certain road, 10 percent of the motorists exceed the posted


speed limit and receive speeding tickets, but 20% of the motorists who
exceed the posted speed limit do not receive speeding tickets. What
percent of the motorists on that road exceed the posted speed limit?

consider the number of motorists as 100. 10% = 10. They speed over and
receive tickets. Also, 20% of these ppl exceed speed limit but no ticket
101.What is the tens' digit and the units'
digit of 123456789^2?
Answer: 21
123456789^2=(123456700+89)^2---
>89^2

102.

Country Chile Haiti China Japan


American GBR Canada Luxemburg France
Per capita 67 40 28 405
102 102 & ; ; ; ; ; ; ; ; ; ; ; ; ; ;
;nbs p; 268 334 & ; ; ; ; ; ; ; ; ; ; ; ;
; ; ;nbs p; &nb sp; 554
water con
-sumption
According the table shown above, which
of the following is right?
I. Japan is more than 10 times of Haiti
II. The median of last five countries is
102
III. The per capita water consumption of
china and Haiti is 34

I. Yes 405 > 10*40


II Wrong...
III Wrong..

Answer: Only I. Is right

103.If x, y and a are numbers, is x > y?


1) x + a > x - a;
2) ax > ay;

Clearly, from I, 2a > 0 or a > 0


From II, x > y, if a> 0. Hence, combining
both the statements, C.
Answer : C

104.If d1, d2, d3, d4 are four different


integers, whether is 0.d1d2+0.d3d4
greater than 1?
1) The least number is 4
2) (0.d1d2)(0.d3d4)>0.5

From I, If 4 is the least number, the sum


will definetly cross 1. Hence is suffcient.
From II, yes, we can determine that the
sum is greater than 1.
For the product to exceed 0.5, the
individual numbers should be atleast >0.6

Hence, the result is D.

105.n+k=0?
1) nk=0
2) n-k=0

From I, we cannot determine the sume.


From II, we still cannot determine.
From I and II, yes we can determine. (n-
k)^2 + 4*nk.
Hence, C.

106.If the product of the three digits of a


three-digit number is 144, what is the
tens' digit of the least such number?

Represent 144 in single digits as 2*9*8.


Hence, the least number has to be 289.
Answer: 8

107.If N is an even number and E is an


odd number, which of the following cannot
be an integer?
A. N/E
B. E/N
C. N/(E+1)
D. (N-1)/E
E. (N+2)/(E-1)

Division of an Odd number by an even


number is not possible.

Answer: B

108.If six people are to be divided to


three groups, each of which contains two
people, how many was are possible?

109.O is the centre of a circle in a


coordinate. The distance between O and
axis-x is greater than or equal to L, the
distance between O and axis-y is M. Does
the circle intercept with axis-x?
1) L>M
2) It intercept with axis-y

From I, Not possible. No references to L


and M.
from II, Not Possible. Only the reference
to M is known.
From I and II, Not possible, since the
distance to x-axis is not known.
hence, E.
110.Someone cost x for something and
the sell price is 10% more than x and final
sold price is 10% less than sell price.
What is percent difference from sold price
to x?

let the cost price = 100. 10% more =


110. 10% less on it = 99.
hence percentage difference = 1 %.

111.A and B use x hour to finish 1000


units and A use 5/3 x hour to finish 1000
units and how many hours, in x, will B use
to finish 1000 units?
3/5x + 1/B = 1/x. 1/B = 1/x - 3/5x =
2/5x. takes 5x/2 hrs.

112.99999^2-1^2=? Answer
10^5(10^5-2) (a-b)(a+b)

113.Water is pumping into a cylinder


container at the rate of 1cube meter per
minute. If the water level rises one meter
per minute, what is the area of the
bottom?

Since, the volume that flows in = 1cu.m,


and height rise is 1meter. the surca area
of the bottom = 1 m.sq

114.One thing cut into 3 pieces, one have


length of ... and other have length of ...,
ask the fraction of the longest one v.s. the
shortest one.

115.The four sides of a parallelogram


have the same length. What¡¯s the length
of one of the diagonals?
1) The side length is 2
2) The length of another diagonal is 2

From I, we do not know if the


parallelogram is a rhombus or square.
From II, same as I.

From I and II, yes we can find out the


lenght of the rhombus . 2^2 = 0.5*2 * D.
D= 4
Answer: C

116.There are 10, 20, and 30 students in


class P, Q and R, respectively. In a certain
test, the average score of the students in
the class P, Q, and R are 79, 85, and 91,
respectively. What's the average score of
the total students?

10 *79 + 20 * 85 + 30 * 91 / 60
Answer: 87

117.Which of the 1/x, x and x^2 is the


least?
1) x>0
2) x<1

From I , we cannot determine.


Since, the fractional value and the
integral values do not concur.
From II, -ve integers and their fractional
values also have to be taken into acc.

From I and II, yes we can safely assume


that they are fractional value and X is the
least.
Answer: C

118.A and B need three hours and six


hour to finish a certain work, respectively.
How many hours will it take for them to
finish half of the work together?

1/3 + 1/6 = 3/6 = 1/2 , 2 hours


Answer: two hours

119.The function T=8h-3t represents the


increased amount of the water in a certain
container, where h is the inputting time
and t is the outputting time during one
day, in minute. What's the inputting time
in a certain day?
1) The increased amount of water is 200
2) In the day, outputting time is 20
minutes

From I or II, we will have two unknowns


in the eqn , hence cannot solve.
With both I and II, we can determine the
value.

Answer: C

120.A sequence is such that,


T(n+1)=Tn/2, then T5=?
1)T3=1/4
2) T1-T5=15/16

From I, yes, we can determine the the


value of T5. that is t4 = 1/8 and t5 =
1/16.

From II, also we can determine the value


of t5, just by replacing T5 successively by
the equation.
t1 - t5=> t1 - t4/2 => t1 - t3/4.. and so
on..

Answer: D

121.During 90 days, a certain beach sold


tickets to the tourists.
There are two kinds of tickets, daily ticket
is $5 each and the seasonal ticket is $200
each. If the total amount of the revenue
on the tickets is $11,000,000, how many
seasonal tickets had been sold?
1) ...
2) A number of ... daily tickets had been
sold.
Answer: D

122.If n=3k, is k an integer?


1) n is an integer
2) n/6 is an integer

From I , it is not possible. N can also be a


prime number and so on.
From II, yes, we can determine. It says
that n/6 is an integer. Hence, n/3 is also
an integer.

Answer: B

123.If x and y are positive, then


(x^1/n*y^1/n)^n=?
Answer: xy

124.Letters A, B, C, C and D are used to


compose a code, such as ABCCD, ABCDC.
How many ways are possible that
between two letters C, there are at least
one another letter?
12, 24, 36, 48, ...
120 ! / 2 !

Answer: 36
a, b, c, c, d: first get the combo. Between
a b d => 3!, then x a x b x d x, then pick
two out these four blank spots to insert c:
Thus p(3, 3) * c(4, 2) = 6 * 6 = 36

125.DS: what is the average of a, b, and


c?
1) a + b + 3c = 17;
2) a + b - c = 1;
Answer: C
Let (a+b+c)/3=v(average ), then
3v+2c=17, 3v-2c=1, v=3

126.DS: a number x = 5*(10^n) + m, m


and n are positive integers and x / 3 has
remainder r, what is r?
1) n = 5
2) m = 1

Consider this part of the equation, 5 * (10


^ n ). this will always lead to 50, 500..
and so on.

Hence, From II, the addition of 1 to 50,


500 or ... will lead to 51,501...
From the divisibility rule, the numbers are
divisible by 3.

Answer: B

127.Machine M can finish a job in 12


hours by itself;
if machine M finishes 2/3 of the same job,
and then machine N, whose work rate is
1/10 of that of machine M finishes the
rest of the job, what is the total amount
of time that took these two machines to
finish the job?
Answer: 48
Let the rate of M to be 1/12, then rate of
N, (1/12)*(1/10)=1/120
(2/3) / (1/12)+ (1/3) / (1/120)=48hr
128.Is x > y?
1) x^2 > y^2
2) y - y^2 > 0

From I, we cannot determine the sign on


the variables. Hence, not possible.
From II, we cannot determine if Y is
positive or not.
From I and II, still not possible.

Answer: E

129.If x, y, a are numbers, is x > y?


1) x + a > x - a
2) ax > ay
Answer: C
1) x + a > x - a => 2a > 0 = > a > 0;
2) ax > ay => x > y if a > 0, x < y if a <
0;
1) and 2) together gives you x > y since
1) tells u a > 0

130.Some girl is buying apples and


oranges; apples costs $0.4 a piece, where
orange costs $0.6 a piece;
if the girl bought 10 of apples and
oranges, and the average cost is 56 cents,
how many oranges does she have to put
back in order to have an average cost of
52 cents for the remaining fruits?

0.4 ( x) + 0.6 ( 10 -x) / 10 = .56; -0.2x +


6 = 5.6;x = 2.

Now, 0.4 * 2 + 0.6 (8-x)/10 -x = 0.52;


0.8 + 4.8 -0.6x = 5.2 - 0.52x;
0.4 = 0.08x; x = 5

Answer: 5

131.A guy sent an order of sofas and


loveseats, if the total cost was $3800,
how many sofas did he buy?
1) One sofa costs $800 and one loveseat
costs $300.
2) There was more than one sofa in the
order.

Not sure of the answer.


From I alone we can determine the
answer, becuase without involving both
the terms the sum does not end up to
3800.

My answer : A.

Given Answer: C

132.A tax of 6 percent is charged to adult


clothing, and no tax is charged for kids
clothing, if a dude bought $149.00 of
clothing, how much did he spend on kids
clothing?
1) The kids clothing cost 43 dollars more
than adults clothing
2) If the same percent of tax is charged
for kids clothing, the total would be ...
Answer: D
1) Let k denote how much he spent on
kids clothing, and a be how much he
spent on adult clothing, then 1) gives the
following:
k + 1.06a = 149
k - 1.06a = 43 => 2k = 192 => k = 96
2) k + 1.06a=149, 1.06k+1.06a=...

133.Two squares x and y, what's the ratio


of the perimeter of x to the perimeter of y
1) One side of x is a diagonal of y
2) The ratio of one side of x to one side of
y is square root of 2

From I, we know that if side of square is


x. the diagonal is x*root2. Hence, we can
determine the ratio.
From II, yes we can determine.
Answer: D

134.You have a rectangle of size 15 feet


by 12 feet, if we want to fit it with
squares with a side of 9 inches, how many
we can fit in such rectangle?
(1 foot = 12 inches)
Answer: 320,
(15*12)*(12*12)/9*9=20*16

135.Someone invested at annual interest


rate r, compound quarterly. What's the
value of r?
1) At the end of the year, the amount of
the interest is between $800 and $850.
2) The interest earned in the second
quarter is $4 more than that in the first
quarter.

From I, it is not possible to determine the


value of r.
From II, we cannot find out the rate, since
we do not know the principle amnt.

My Answer : E

Given Answer: B

136.One apple is 40 cents, and one


orange is 60 cents. Now J bought total 10
apples and oranges and the average mean
is 56 cent. If J wants to keep the average
mean 52c, how many oranges should be
taken out?
My answer is 5.

137.Yesterday each of the 35 members of


a certain task force spent some time
working on project P. The graph shows
the number of hours and the number of
members who spent that number of hours
working on project P yesterday. What
was the median number of hours that the
members of the task force spent working
on project P yesterday?
A. 2
B. 3
C. 4
D. 5
E. 6
Answer: E

138.A sum of $1000 was invested at the R


percent semi-annual interest,
compounded. If at the end of one year the
total interest is $80.56, what is the
possible value of R?
A 5<R<6.
B 6<R<7
C 7<R<8
D 8<R<9
E 9<R<10
Because the interest income is slightly
above 80.00, then pick 8% first, semi-
annual compound. You will find the
interest income will be 81.6, $1.04 higher
than 80.56. So the choice must be just
slightly lower than 8%. So choose C.

139.40% members of a community are


women. Out of this 30% of the women
are 40 years or more. There are 1800
men. How many women are 40 years or
more?

1800 is 60%. 40 % = 1200. out of this


30% = 360

140.We need to find the area of the


figure. ABC=60 and ADC = 90 and
DAB = DCB

141.What is the range of the numbers 2,


3, 4, 6, 7, 9, 10, 12, 20, and x?
1) The mean is 9
2) x is the median

Answer :D

If the mean is 9, we can determine the


number x.
From II, if x is the median, we can
determine x, from the given series.

hence, D.

142.PS: (x-y)^2+2*y^2=27, x=?


Answer: 6

Did not understand the question. There


are two unknowns.. so how to determine
x?

143.(47.5/0.0475)^3=?
Answer: 10^9

144.The average value of five numbers is


6, is the standard deviation greater than
10?
1) Four of the numbers are 16
2) One of the numbers is ¨C4

From I, we can calculate the whole


series..
16,16,16,16,x. since the avg is known, 30
= 64 + x; x = -34
Hence,we can determine the median of
the series.

From II, i cannot make out anything..

Answer: A

145.One box contains 30 purple balls and


20 red balls, another box contains 15
purple balls and 35 red balls, if Wendy
randomly chooses one box and draws one
ball from it, what will be the possibility
that the ball will be purple?
The probability that eighter of the bags is
picked is (1/2).
Hence, the probablilty of picking the ball
from 1st bag + probability of picking the
boll from 2nd bag.

Answer:
(1/2)*(30/50)+(1/2)*(15/50)=9/20

146.M, N and K are integers. Is the


product of them divisible by 3?
1) The sum of them is divisible by 3
2) M, N, and K are consecutive integers.

Answer: B
1) 5, 8, 11

147.If An= A(n-3)+18, the first several


numbers are 102,108,114, which of the
following numbers is in the sequence?

the above is a progression, with d = 6 and


a = 102

Answer: 594
An=102+6n
148.If line m is perpendicular to line k, is
line k intersects to line x=-1?
1) The slope of line m is 3
2) Line k (or m?) passes through the point
(0,4)
We must visualize that x=-1 is a vertical
line with slope of infinity.
hence, any line with a defined slope will
intersect the line.

From II, we cannot determine the slope of


the line and hence, we can not confirm.

Answer: A

149.There are three kinds of bonus, each


is $735, $150, $75. The company will give
bonus to its employees. Each bonus has
to be signed to at least one person. Now
the company has the total bonus of
$64,800. What is the minimum number of
employees who can get the bonus?

Since, min ppl have to get the profits,


max profits need to be given out...

Hence, number of $735 = 88.


Remaining is $120, which can be given to
1 person, hence, the total number of ppl
to whom profits can be given is 89.

Something wrong with the Answers


options.

A.10 B.11 C.12 D.13 E.14

150.If x is positive, is x<1?


1) x<x^1/3
2) x^3<x

A positive number is smaller than itself


when it is squared is when it is under 1.

From II, we can determine the same


information.

Answer: D

151.It is same to Q104


If d1, d2, d3, d4 are four different
integers, whether is 0.d1d2+0.d3d4
greater than 1?
1) The least number is 4
2) (0.d1d2)(0.d3d4)>0.5
Answer: D
1) Let them be 4, 5, 6, 7, then both
0.47+0.56 and 0.46+0.57 are greater
than 1
2) Let them be a, b, c, d
Then ab*cd/100*100>1/2
ab*cd>5000
ab+cd>/=2*(ab*cd)^1/2---
>ab+cd>2*5000^1/2=100*2^1 /2 =141

0.ab+0.cd>1.41

152.The total radio and camera sold in a


store is about N, how many radios was
sold?
1) The radio's mean price is N1
2) The camera's price is N2

From I, we cannot determine, eighter of


the two numbers.. hence, not possible.

Same for II,


Hence, E.

Discussions : MJJ 152-164

153.DS: X<5?
1). |x-5|>0
2). x^2 + x <5

From I, we cannot determine if x<5. If X


is 4 or 6, we still get an answer of 1.
from II, it solves as x(x+1) <5, in both
ways, x is eighter 0 or less than 4. Hence,
B.

Answer: B

Discussions : MJJ 152-164

154.DS: x is a prime, x=?


1) The sum of the factors of x is 24
2) The product of the factors of x is 23

From I, we know that x is prime, whose


factors are itself and 1, hence, we can tell
that the number is 23.
from II, we can definetly tell that the
prime number is 23.

hence, D.

Answer: D
Discussions : MJJ 152-164

155.Of a group of 200 people, how many


are male and with brown hair?
1) 50 percent of the people are brown
hair
2) 60 of the people are male

From I , we can determine only the total


number of ppl with brown hair, which also
contains women.
from II, we can tell the number of males
in the grp.

From both, we still cannot find out the


number, since we do not know the
number of women with brown hair.

Answer: E
Discussions : MJJ 152-164

156.If [X] is defined to be the number


greater than x, y>-3£¿
1) [y/2]=0
2) [y/4]=0
Answer: E
1) [y/2]=0 >>> 0>y/2 &nbs p;
2) [y/4]=0 >>> 0>y/4

Discussions : MJJ 152-164

157.n+k=0?
1) nk=0
2) n-k=0

from I, we cannot tell , if n or k is zero.


From II, we can tell than n and K are
same.

From Both, it is infered that, both n and k


are zero.
Hence, C.

Answer: C
Discussions : MJJ 152-164

158.A rectangle's dimension is x by y,


where x<y. If the perimeter is x+16 and
areas is 24, x=?
Answer: x=4
Perimeter = 2x + 2y = x + 16 => x + 2y
= 16;
Area= x*y = 24 => x = 24/y, => 24/y +
2y = 16, 24 + 2y^2 - 16y = 0, y^2 - 8y
+ 12 = 0 => (y - 6)(y -2) = 0 => y = 6,
or y = 2, since x < y, then y has to be 6,
thus x = 24/6 = 4

Discussions : MJJ 152-164

159.Line L passes through point (1,4),


and the product of its intercepts with axis-
x and axis-y is negative. Which of the
following point is on the line L?
A. (2,3)
B. (5,7)
C. (4,2)
D. (4,-1)
E. (5, -1)
Answer:
When u draw it out, the line has to go
through either positive-x and negative y,
or positive y and negative x, only (5, 7)
makes this happen, thus it's 2)

Discussions : MJJ 152-164

160.If (0.00048*10^n)/19200<2*10^10,
what is the greatest possible value of n?
Answer: 17
Rethink the problem as 0.0005 * 10^n /
20000 < 2*10^10 => .0005 * 10^n <
4*10^14 => 5* 10^(n-4) < 4 * 10^14,
thus n - 4 = 13 => n = 17

Discussions : MJJ 152-164

161.Sequence a1, a2, a3 ... is such that,


a3=a2+a1,a4=a1+a2+a3, ... If an=P,
a(n+2)=?
Answer: 4P
a(n+2)=a(n+1)+an+a(n-1)+...
a(n+1)=an+a(n-1)+...
an=P=a(n-1)+...
Then, a(n+2)=4P

Discussions : MJJ 152-164

162.If one man, two children, one woman


are to be selected from eight men, ten
children, ten women, respectively, how
many ways are possible?
Answer: 3600
C1,8*C2,10*C1,10=3600

Discussions : MJJ 152-164

163.5^21*4^11=2*10^n, n=?
Answer:21

Discussions : MJJ 152-164

164.Is the least element of A less than


that of B?
1) The range of A is bigger than that of B
2) The biggest element of A is smaller
than that of B
Answer: C
A2-A1>B2-B1, A2<B2, >>>A1<B1
1) A's range is bigger than B's range, but
since A and B could be anything, it is not
sufficient to establish the relationship
between the smallest elements in each A
and B, thus 1) is not sufficient
2) The biggest element in A is smaller
than the biggest element in B, since we
know nothing of the range of the two sets,
it's not sufficient either
1) and 2), we know A's range > B's range,
a(the biggest in A) < b(the biggest in B),
since A's range = the biggest - the
smallest, if A's range > B's range => the
biggest in A - the smallest in A > the
biggest in B - the smallest in B, since a <
b then the smallest in A has to be less
than the smallest in B, thus 1) and 2)
together are sufficient
Therefore the answer is C

Discussions : MJJ 152-164

165.Of some five numbers, the average is


10, mode is 15, the greatest number is
equal to the mode, and the median is 12.
What is the least number?
1, 9, 13, ...
Answer: 1

From the information, lets make a


series... No1, No2, 12,No3, 15.
Since, least numer is asked, lets plugin
ans check.
Also, mode is 15, hence, 15 should occue
atleast twice.

Let the series be 1, No2, 12,15, 15.

The sum is 43. For the avg to be 10, the


sum should be 50.The second number is
7. Hence, the only possible least number
is 1.

Discussions: MJJ 165-174

166.DS : x>y?
1) x^2>y^3
2) x^3>y^4

From I, we cannot determine if x is


negative or not, and still x^2 > y^3.
Eg:- x = -4, y = 2, x<y, but x^2 = 16 >
y^3..

Discussions: MJJ 165-174

From II, not sufficient, hence, not


possible.

From I and II, still not possible.

Answer: E

167.In one hour, H can produce three


dozen of something and T can produce
four dozen of something. If they took 14
hours to produce 77 dozen of the product,
what is the least simultaneous working
hours? (It means that, during the 14
hours of period, T and H worked
simultaneously for some time, then
divided the other time.)
Answer: 7
Let a to be the H's hour, b to be the T's
time, c to be the common time.
3a+4b+(3+4)c=77 and a+b+c=14
=>3a+4(14-a-c)+7c=77 = =>3c=a+21
When a =0, the c has the least time, 7

Discussions: MJJ 165-174

168. P---Q---------------R---S
The locations of four points in the number
line is shown above. If PR=OS, PR=5PQ, P
is 1/3, S is 1/2, what is R?
Answer: 17/36
pr = qs, rp = 5 * pq
Since ps = pr + rs = pq + qs, and pr = qs
=> rs = pq, let rs = pq = x, then since pr
= 5x, then ps = 5x + x = 6x = 1/2 - 1/3
= 1/6 => x = 1/36, thus r = s - 1/36 =
1/2 - 1/36 =17/36
Discussions: MJJ 165-174

169.If An=200+0.2*A(n-1), and A1=200,


A50 will fall in which of the following
regions?
(230,239), (240,250), ...
Answer: (240,250)
A(n)=200+0.2*A(n-1),A(n-
1)=200+0.2*A(n-2)=>
A(n)-A(n-1)=0.2[A(n-1)-A(n-2)]=0.2^(n-
1)(A2-A1)=0.2^(n-1)*40=> A50-
A49=0.2^49*40 and
A50=200+0.2*A49=>
A50=200/0.8-0.2^49*40=250-
10x(0.2^49)
Therefore, 240<A50<250

Discussions: MJJ 165-174

170.Someone invested a certain amount


of money at the 2.5 percent annual
interest rate. At the end of the fifth year,
the total amount in the account is
$11250. How much did he invest?
The number maybe inaccurate, pay
attention on the thinking
x*(1+2.5%)^5=11250
x+12.5%*x+10*2.5%^2x+...(can be
neglected)=11250
Above all: x*(1+2.5%)^5 approximately
equal to x+12.5%*x
Discussions: MJJ 165-174

171.About a certain kind of records of a


whether station. Which of the least values
in July and in August is bigger?
1) The range of the records in July is
greater than that in August.
2) The greatest record in July is less than
that in August

From I, we cannot derive information on


which has a greater value.
From II, we can determine that the
greatest record is in August.Also, But,
does not tell which month has the greater
lowest value.
From I and II, it is clear that August has
the greater least value, since the range of
records in July is more than in August and
August values are higher.

Answer: C

Discussions: MJJ 165-174

172.A circle, which circumference is 1


mile, is equally divided by nine points O,
P, Q, R, S, T, U, V, and W. At the speed of
15mile/hour, someone started out from
the point O. After three minutes, which
region did he arrive in?

Since , the circumference is 1 mile, each


section is 1/10 mile. distance travelled for
3 min at 15 mile/hr is 3* 15/60 = 3/4.
3/4 = 7.5/10, hence, in the 7th section,
which is UV.

Answer: UV

Discussions: MJJ 165-174


173.9^x + 9^(-x) = 62, 3^x + 3^(-x) =?
Answer: 8
[3^x + 3^(-x)]^2=9^x + 9^(-
x)+2=62+2=64 >>> 3^x + 3^(-x) =8

174.One of the angles of a pentagon is 80


degree. If the other four angles have the
equal measure, what's it?
Answer: (n-2)*180-80/4, n = 5 , 540 - 80
/ 4, ans = 115.

Discussions: MJJ 165-174

175.PS: invest 200,000, interest 5%,


annual compounding, then after one year
invest this investment and the interest
earned to a two yrs program, interest is
y%, compounded annually, after two yrs,
the interest is 8484, ask what is y?
A. 2.0, B2.5 C. 3.0 D.3.5 E4.0

P = 200,000; A = p(1.05) = 200,000


+10000 = 210,000 .
210000(1+y/100)^2 = 8484
By plugin method, A is the answer, y =
2%.

Answer: A

Discussions:MJJ 175-182

176.DS: x and y are integers, and when


x+y divided by 11, the remainder is r, ask
for r:
1) When x-2 is divided by 11, the
remainder is ...
2) When 35-y is divided by 11, the
remainder is 1 (I am not sure)
Answer is C.

Discussions:MJJ 175-182

177.There are 6 students, and they are


chosen to form 3 groups to make
presentation. Each group will be assigned
Asia, Africa or America as their
presentation subject. And the question is
how many groups could be formed.
The first 2 ppl can be chosen in 6C2 ways.
the next 2 in 4C2, and other 2 in 1 way.

Answer: C(2, 6)*C(2,4)*C(2,2)=90.

Discussions:MJJ 175-182

178.If 0.02<X- root10<0.021, the value


of (1/root10 ¨C1/x) is most close to which
of the following numbers?
A.0.2 B.0.02 C.0.002 D.0.0002
E.0.00002
Answer: C
1/root10 ¨C1/x=(x-root10)/x*root10
Because x approximates root 10,
therefore (x-root10)/x*root10
approximates (x-root10)/10= => 0.002

Discussions:MJJ 175-182

179.X= 15 (0.0001) (0.00001) = a


(10)^b; 0.1<a<1, b =?

The main idea is to equal the number of


decimal places in the LHS.
Answer: -7
15 (0.0001) (0.00001)=0.15x(10^-7)=a
(10)^b

Discussions:MJJ 175-182

180.DS: S0 =0, S10 =?


1) Si = i + Si-1
2) Si + Si-1 = i^2

From I, we can perform iteration and find


the value of S10, s10 = 10 + S9;s9= 9 +
S8.. and so on
From II, same as I. We can calculate the
same.

In GMAT, for such question, we need to


see if this pattern can be traced back to
the given value!!

Discussions:MJJ 175-182
Answer: D

181.P is an integer, is P a prime number?


1) P+3 is a prime number
2) P^2+3 is a prime number

From I, we can determine that, if P+3 is a


prime number, P cannot be a Prime
Number.Sine, to be a Prime number, the
number should be an odd number.
But, we have an exception,.ie, 2. Hence,
Not possible.

From II, P^2 +3 is a prime number, P^2


is an even number, hence, P cannot be
prime. Again, 2 is an exception. Hence,
Not possible.

From I and II, the only possibility is 2.


Hence, Possible.

Answer: C

PS: For all Prime Number calculation,


pluggin 2 first..

Discussions:MJJ 175-182

182.DS: x^2+bx+c=0, r and s are the


solutions of the formula. Is r*s<0?
1) b<0
2) c<0
Answer: B
X^2 + BX + C = 0, (X-R)(X-S)=X^2 + BX
+ C=0
Then B= - (R+S), C=RS
From statement 2, we know that c<0,
R*S<0

Discussions:MJJ 175-182

183.If x is one of the a group of 21


numbers and is four times the average of
other 20 numbers, x is what fraction of
the sum of all the numbers?

If X is one of the 21 numbers, and is


equal to 4 times the average of other 20
numbers, then

x = 4A, where A = avg of the remaining


20 numbers.

Sum of all numbers is x + 20A= 24A;


Hence, the 4a/24a = 1/6

Answer: 1/6

Discussion: MJJ183-188

184.The line represented by which of the


following equations is perpendicular to the
line represented by y=5x?
y=-5x, y=-x/5, y=x/5,...

The product of 2 perpendicular lines


should be -1. the slope of the given line is
5. So we need to select a line with slope
of -1/5.

Which is y=-x/5.
Discussion: MJJ183-188

185.Three of the 15 books are defect. If


two books are to be selected from the
book, what is the probability that both
two books are not defect?

Number of ways you can pick 2 books


from 15 = 15C2.
Number of ways to pick books without
defect = 12C2. Probability = 12C2/15C2.
Discussion: MJJ183-188

186.If xy=165, and x>y, the least


possible value of x-y is:
2, 4, 8, 23, 15

After factorizing 165, we get 11 and 15,


hence, the least difference is 4.
Discussion: MJJ183-188
187.In a certain class, the range of boy's
heights is 14 inches and the range of the
girl's heights is 12 inches. What is the
range of the heights of all the students?
1) The shortest boy is 3 inches shorter
than the tallest girl
2) The height of the tallest boy is 74
inches

From I, we can determine the range which


is 14 + 12 - 3.

From II, we cannot determine the girls


height from this, hence, we cannot.

Answer is A.
Discussion: MJJ183-188

188.A person had two accounts. He


invested $1000 in an account paying 4%
interest annually and $2000 in another
account paying 2% interest annually.
Another person had only one account to
which he invested $3000. If they invested
at the same time, how much should the
second person's rate be, so that in the
end of one year, they earn the same
amount of interest?

person A, interest = 1000*4/100 +


2000*2/100 = 40 + 40 = 80.
person B, 80 = 3000*r/100; r = 8/3 %

Discussion: MJJ183-188

189.Set A contains five negative numbers.


Set B contains six numbers, one of which
is 0 and three of which are positive. If one
number is selected from A and another is
selected from B, what is the probability
that the product of them is negative?

From set B, the probability that a -ve


number is always picked from A is 1.
from set A, the probability that a +ve
number is picked other than 0 is 3/6 =
1/2.

Probability = 1/2.

Answer: 1/2

Discussion:MJJ 189-199

190.6^(a+4)/[2^(a+2) * 3^(a-2)]
(2^a *3^a * 6^4)/(2^a*3^a *
2^2*3^(-2) )
=6^4*9/4

Discussion:MJJ 189-199

191.The sum of 1/41, 1/42, 1/43, and


1/44 is between:
Answer: 4/41~4/44

Discussion:MJJ 189-199

192.A person plans to decorate his house.


Company A's service price is $2.4 per
square meter. Company B's service price
is $3 for each of the first 500 square
meters and $2 for each square meter
there after. Which company's service is
more economical?
1) The total area of the house is greater
than 1000 square meters
2) The total area of the house is less than
1200 square meters

from I, we cannot determine the actual


price, since, no upper limit is mentioned.
From II, whatever price is considered
within 1200, we get consistent values....

hence, B.

Discussion:MJJ 189-199

193.If n is a positive integer, is


n*(n+1)*(n+2) divisible by 12? [or, is 12
divisible by n*(n+1)*(n+2)?]
1) n*(n+2) is even
2) n+1 is even

From the question, we know that the


series is always divisible by 3. So, to
prove that 12 is either divisible or vice
versa..
we need to know if there is an even
number in it.

From I, yes, we can determine.


From II, yes, we can determine.

The author's answer is D


The question maybe is incomplete.

Discussion:MJJ 189-199

194.A person loaned $2000 from a bank


for two years. In the end of every six
months, he paid 5% interest first, and
then paid $500 back to the bank. When
he paid off all the money he loaned, how
much interest had he paid?
Answer:
2000*5%+1500*5%+1000*5%+500*5%

Discussion:MJJ 189-199

195.DS. x/y < b/c ?


1). cx-yb<0
2). bx-cy<0

From I, we can determine the equation by


rearranging it.
cx<yb; x/y < b/c
From II, we cannot dermine.

Hence, A

Discussion:MJJ 189-199

196.PS. x is a positive integer. If both the


units' digit of (x+1)^2 and the units' digit
of (x+3)^2 are 1, what is the units' digit
of x-3?
X = 8.
Answer: 5

Discussion:MJJ 189-199

197.The formula for Celsius degree


exchanging to Fahrenheit degree is
F=9/5X+32. If five data measured with
Celsius way are A, B, C, D, and E, where
A<B<C<C<E, when they are exchanged
to Fahrenheit degree, the range will be:
MY ANS: 9/5(E-A)

Discussion:MJJ 189-199

198.If two of the sides of an isosceles


triangle are 6 and 6*root 3 long,
respectively, what is the perimeter of the
triangle?
1) One of the angles is greater than 90
degree
2) The perimeter is less than 24

From I, if one of the angeles is greater


than 90, the sides along which the angle
is based has to be short, which is 6.
hence, the perimeter is 6+6root3

from II, it is said that the perimeter is less


than 24. The only other option is
6+6+6*root3

Discussion:MJJ 189-199
Answer: D

199.Of seven consecutive integers, how


many are multiple of 6?
1) The median is the multiple of 6
2) The sum of the numbers is the multiple
of 6

From I, we can tell that 3 numbers on


eighter side of the number series is not a
multiple of 6.
From II, by plug-in method we can
determine.

Answer: D
Discussion:MJJ 189-199

Questions that appeared in my GMAT


today(08/18):

200. Six students form 3 pairs and they


have to be assigned to 3 different
continents. How many ways can they be
assigned.

Answer: 6C2 * 3! = 90 (Discussed


earlier).

201. If r*s = s*r, which of the following


are correct.

I) r+s-rs
II) r/s + s/r
III) cant remember. but III is not the rite
choice.

I&II are the correct.

202. Given the coordinates of a triangle in


xy plane. AB parallel to x axis and A
coordinates (a,b), B coordinates (6a,b), C
coordinates (c,4b)

A) a = 3/4b
B) Area of the triangle is 90 units.

I have selected E but I guess its C.

203. Out of X students, 64% are


educated, 48% are educated males. What
is the fraction of female that are
educated.

Ans: 16/64 = 25%.

204. An amount 150K invested in 1984


grows X% in 1985 and Y% of 1985 in
1986. What is the difference b/w amounts
in 1986 and 1984.
A) X + Y = blabla....
B) (1+x/100)(1+y/100) = given amount.

Ans: B.

205.A sales person wants to start five cars


with their keys, but he doesn't know the
right keys to them. If he starts the five
cars with the five keys randomly, which of
the following cannot be the number of the
cars that he exactly starts?
A.0
B.2
C.3
D.4
E.5

If you are able to start 4 cars, then


automatically, the 5th will start, since the
only other key left, will start the last car.
Hence, the answer is 4.

Answer: 4

Discussions: MJJ 205-215

206.3x-2y =?
1) x/0.2 - y/0.3 = 9
2) x/0.3 - y/0.2 = 7

From I, we can derive the same equation


that is asked for.
From II, it is not possible to derive the
same equation.

Hence, A.
Discussions: MJJ 205-215

207.If 15% of the plants have fungus and


die, and 40% of plants with fungus did
not die. What percent of plants has
fungus?

Confusing... Not clear...


Discussions: MJJ 205-215

208.There are five pairs of shoes with


different colors in a box. If two shoes are
to be selected from the box without
replacement, what is the probability that
they have the same color?
The number of ways that you can pick 2
shoes from 10 is 10C2.
The number of ways that you can pick 2
shoes with same color is 5 ( 5 pairs)

probability is 5/10C2

Answer: 5/C(10,2) = 1/9

Discussions: MJJ 205-215

209.If [(2X)^(1/2)]*[(3W)^(1/4)] =
[(2Y)^(1/2)]*[(3Z)^(1/4)], and X=4Y, W
is what fraction of Z?
Answer: 1/16
Discussions: MJJ 205-215

210.In a certain store, the original sale


price for something is X% of the retail
price. One month later, the discount sale
price is Y% of the original sale price. If
the retail price is 72, what is the discount
sale price?
1) X*Y = 5600
2) 8X = 7Y

From I, we can calculate the discount


sale.
the sale price is retail price *x. Now, the
discount price is retailPrice*x*y.
Retail price is given and product is given.

From II, we can only get teh ration of the


%. Not usefull
Hence, A.

My answer: A
Discussions: MJJ 205-215

211.If x is an even positive integer, which


of the following is (are) the remainder of
(3x)/5?
I.0, II.1, III.2

ex, 15,21,12
Answer: all are right

Discussions: MJJ 205-215

212.If C(3,5)=C(x,5), x=? (x is not 3)


Need not calulate... C(r,n) = C( n-r,n)

Answer: x=2

Discussions: MJJ 205-215


213.If x*(2^y)=176 and x is a prime
number, x*y=?

From the above , we notice that 176 is a


factor of 11. The remaining is 4. Hence,
the product 44.

Answer: 44

Discussions: MJJ 205-215

214.In a xy-plane, a line passes through


point (0,4) and (4,0), which equation
represents the line?

My Answer: x/4 + y/4 = 1


Discussions: MJJ 205-215

215.The length of a certain nail differs


from the average length of the entire lot
of nails in amount between d and 2d
(note: either shorter or longer, the
difference is between d and 2d) ask if the
length of this certain nail is > 75
1) d= 4
2) Mean length = 80

From I, we cannot determine anything.


From II, we still cannot determine
anything.

From I and II, the range is between 76


and 72. Still cannot determine.

Answer: E
Discussions: MJJ 205-215

216.Paul has x dollars. Tom's money


equals Paul's plus twice the Anita's. If Paul
and Anita cost one dollar, respectively,
Paul's money will twice Anita's. How much
money, in x, does Tom have?
Answer: T=2X+1
Another version:
Paul has m dollars. Tom's money is twice
the sum of Paul's and Anita's. If Paul and
Anita cost three dollar, respectively, Paul's
money will twice Anita's. How much
money, in m, does Tom have?
Answer: 3m+3

217.Bruce bought only $0.32 stamps and


$0.23 stamps. How many $0.23 stamps
did he buy?
1) He bought $0.87 worth of stamps.
2) ...

If total number of stamps or the relative


difference between the number of stamps
is given, its C.
Else, E.
218.Is x^2-y an integer?
1) Both x and y are integers
2) x*y is negative

From I, yes we can determine if the


difference is -ve, since, both are integers,
the difference will also be negative.

From II, we do not know if eighter is not


an integer.

My Answer : A

GivenAnswer: E

219.If 140*n is the square of an integer,


where n is an integer. What is the least
value of n?

140 = 10 * 14; 2 *5 *7*5. Hence, to


make it a complete square , we need 35.
Answer: 35

220.From 1990 to 1993, the sales value


increased by 40%. From 1990 to 1995,
the sales value increased by 40%. What
was the percent increase from 1993 to
1995?
Answer: 29%
I am not getting the same answer..

221.A contains two red flowers and x


white flowers; B contains five red flowers
and three white flowers. X=?
1) If one flower is selected from A and B,
respectively, the probability that both two
flowers are red is 1/4
2) If one flower is selected from A and B,
respectively, the probability that both two
flowers are white is 9/40

From I, yes, we can determine the


number of white flowers.

the prob is (1/2)*[ x/x+2 + 5/8 ] = 1/4.

Similarly, From II, we can find out the


number of white flowers......

hence, D

222.If 10^x*(0.0099999)>10^3, x at
least is: 6

223.If the symbol * represents either


addition, subtraction, multiplication, or
division, what is the value of 6 * 2?
1). 10 * 5 = 2
2). 4 * 2 = 2

From I, we can determine that *


represents Division.
From II, we cannot determine whether *
is subtraction or division.
Hence, A.

Discussions: MJJ 223-235

224.In town X, 64% of the population are


employed, and 58% of the population are
employed male. What is the percentage of
the female that are employed?
A .16% ...D.32% ...

Answer : Humanly impossible !!

Discussions: MJJ 223-235

225.If r@s=s@r which of the following


can define the equation?
1. r@s=r+s-rs
2. r@s=(r+s)(r-s)
3. r@s =r/s+ s/r

Its 1 and 3
2 is ruled out. If eighter is negative (r-s)
part of it will eighter be -ve or positive,
which will not yield the same result.

eg, r=2,s=-3 r@s = (-1)(5) = -5


s@r = (-1)(-5) = 5. Hence, 2 is ruled
out,

Discussions: MJJ 223-235

226.What is the equation for a line which


passes (30,0) and (0,20)?

(y - 20)/x = 0-20/30 = -2/3 ; -2x = 3y -


60; 2x + 3y = 60

Discussions: MJJ 223-235

227.What is the average of 59


consecutive integers?
1) The sum of the largest number and the
least number is 1
2) The sum of all is 1

From I, we know that the sum of largest


and lowest number will be negative only if
either is negative but not both.
Here, the largest being 29 and the lowest
being -28. The sum is 1. So , we know the
series and hence, can find the average.
From II, if we know the sum of all the
numbers, the avg will be 1/59. Hence,
enuf.

Hence, D.
Answer: D
Discussions: MJJ 223-235

228.The sales price of a picture is 10%


more than the cost to it. In a promotion,
the discounted price is 10% lees than the
original price. What is the relation
between the discounted price and the
original price?

Let us consider the price of the item as


100.
Sales price = 110
the discounted price is 10% lees than the
original price
Discounted price = 90

hence, the relation is 10/100 = 10% less

If the discounted price is 10% lees than


the sale price
Discounted Price = 110 -11 = 99.

Then, the relation is 1% less

Given Answer: discounted price is 1% less


(not more) than the original price.

Discussions: MJJ 223-235

229.DS. (x+y)^2=?
1) x=y-3
2) x and y are prime numbers.

Only the difference of 5 and 2 is 3


Answer: C

Discussions: MJJ 223-235

230.What is the relationship between


km/h and m/second?
1 KM/HR = 5/18 m/sec

Discussions: MJJ 223-235

231.Someone invested $20,000 at x


percent annual interest and $25,000 at y
percent annual interest. At the end of the
first year, how much interest did he earn
from the $20000 investment?
1) The interest from $20,000 is $300
more than that from $25,000
2) x=(2/3) y
Answer: C

From I, we can only get the relation


between the two folrmulas. We will still
have two vars X and Y.
From II, we will get the relation between
the two interest rates, still the interests
earned by the two is not known.

From Both, we can realate the above


unknowns and get a solution.
hence, C

Discussions: MJJ 223-235

232.Of the members in a certain group,


62% are shareholders and 47% are
employees. How much percent of the
members are shareholders but not
employees?
38%, 45%, 53%...

100=62+47-(both)
both = 9%
So Only S = 62-9=53%

Discussions: MJJ 223-235

233.A circle is divided to six sectors


equally, and another circle is divided to
eight sectors equally. What is the ratio of
the measure degrees of their central
angles?

6 divisions: angle of each sector is 360/6


= 60 deg
8 div: angle of each sector is 360/8 = 45
deg
ration of 6 sec/8 sec = 4/3

Discussions: MJJ 223-235

234.(x^1/n*y^1/n)^n=?

=> (xy)^1/n*n = xy
Ans:xy

Discussions: MJJ 223-235

235.Three machines have the identical


rate. If two of them work together, it
takes k hours to complete a certain work.
If all of them work together, it takes two
hours less to complete the same work.
K=?

Let the rate of each machine be x. Rate at


which each machine works in one hour is
1/x.

In K hours, they work k/x.

Two machines, will work at 2k/x.

With three machines, its 3(k-2)/x. Since


the work done is same,

3(k-2)/x = 2k/x; 3k -2k = 6; k=6.

Discussions: MJJ 223-235

236.A plant can be green or yellow, and it


leaf can grow to be curve or straight. If
the probability that grow to be green is x,
to be straight leaf is 1/4, and to be green
and curve leaf is 3/8, what's x?

Probability to grow straight leaf = 1/4


Probability to grow curly leaf = 3/4

Probability to grow green and curly =


x*3/4 = 3/8;
x = 1/2

Discussion: MJJ 236-243

237.DS: m and n are integers, is m^n an


integer?
1) n^m is positive
2) n^m is an integer

if m is negative in eighter case, we get a


fraction. Hence, not integers.
Thus , cannot be solved.

Answer: E

Discussion: MJJ 236-243

238.If N is an even number and E is an


odd number, which of the following cannot
be an integer?
A. N/E
B. E/N
C. N/(E+1)
D. (N-1)/E
E. (N+2)/(E-1)

Ans: B. You cannot divide an ODD number


by even number

Discussion: MJJ 236-243

239.In a company conference, each


female employee carries three pens and
each male employee carries two pens.
How many female employees attend the
conference?
1) The total number of the pens is 26
2) The total number of the employees is
...

From I, 3x + 2y = 26. Not possible to


solve
From II, x + y = Z.. not possible

From I and II, we can determine the


values of X and Y.

Answer: C

Discussion: MJJ 236-243

240.A simple but tricky question:


C5,12=12*11*2*3*
Answer:
B:8*9*11*

Discussion: MJJ 236-243

241.If x>0, is y>0?


1). x>7/2y
2). x>|y|

From I, we cannot determine the value of


y
From II, Not possible to detect, becuse
either way x may be greater than Y. i.e, if
y were -2 or 2, x is greter than both
values.
Answer : E

Given Answer:I am not sure about


statement 2, chose E

Discussion: MJJ 236-243

242.Three machines in a coalmine are


used for transporting coals. X can
transport 1/4 of a certain amount of coal
in 4 hours, Y transport 1/2 in six hours,
and Z transport 1/3 in eight hours. If they
work together for 24 hours, how much will
they transport?

X can transport in 1 hr : 1/16


Y in : 1/12 and Z in:1/24
total work in 1 r = 1/16 + 1/12 + 1/24
In 24 hr = 3/2 + 2 + 1 = 9/2 = 4.5

Discussion: MJJ 236-243


243.The average of the data in 12 weeks
in a certain company is n. If 24, the data
in a certain week, is replaced by a
number, the average became n-3/4. What
is the number?
15,18,23,24,33
Answer: 15

12n - 24 + X/12 = n-3/4


n - 2 + X/12 = n -3/4; X/12 = 5/4; X =
15

Discussion: MJJ 236-243

244.If the slope of line represented by ax-


by=c is 2/3, what is the value of b?
1) a = 4
2) c=...

The slope is given by -a/b.

From I, we can determine b, since slope


and a is given.

From II,we cannot determine the value of


b.
Answer: A

Discussions: MJJ 244 - 250

245.Working together at the identical


rates, three people can complete a certain
work in 36 hours. If another additional
person, at the same rate, working
together with them, how many hours will
be decreased to complete the work?

Let x be the work that is done by each


person, then
3/x = 1/36; x = 108 hrs.
4/108 = 1/27; 27 hours.
The difference is 9.

Answer: 9
Discussions: MJJ 244 - 250

246.4^x+2^2x=2^200, x=?
Answer: x=199/2

247.DS: Which of 1/x, x, x^2 has the


least value?
1) x>0
2) x<1

From I alone, we cannot determine the


OA.

from II, we do not know in which range


does x lie.. 0 to 1 or 0 to -infinity.

From I and II, we know the range in


which x lies, and hence, can determine.
Answer: C
Discussions: MJJ 244 - 250

248.A store sells cake mix, muffin mix


and bread mix. Of the 100 customers
purchased mix of the store, 50 purchased
at least cake mix, 40 purchased at least
muffin mix, 20 purchased both cake mix
and muffin mix. How many customers
purchased only bread mix?

the total number if people = 100


Total number of people who bought cake
mix = 50
Total number of people who bought
muffins = 40
Both = 20

so, 100 = 50 + 40 -20 + only bread;


bread = 30

Answer: 100-¨50+40-20£©=30
Discussions: MJJ 244 - 250

249.A group cruise contains n people, n is


between 50 and 150. What is the total
cost of the group?
1) The total cost of the group is (300n-
n^2) dollars
2) Each person spent $160.
My answer: C

From I or II, we cannot determine the


numbe of persons,

From I and II, we determine the equation


300n-n^2=160n, from which we can
determine n.

Hence, C.
Discussions: MJJ 244 - 250

250.If x, y, w, z are integer and


x+y=w+z, which of the following about
which of the numbers is (are) even is
right?
a) None, ...
d) None, two, four
e) None, one, two, three, four
Incomplete Question
Discussions: MJJ 244 - 250

Given Ans: D

Edited by Spiderman on 04 September 2005 at


4:47am

Back to Top

meher13
VIP Posted: 04 August 2005 at 2:43pm | IP Logged
21. Ans is 128 and not 120. Convert the hrs to mins and back to hrs to
ease the fraction

Joined: 14 October 2004 22. I agree with the point for E. But and also will add that it will be E still
India because we do not know the extent of AB<DE and BC>EF. Hence, cannot
Posts: 27 say for sure the conclusion
Gender: Not Specified
23. E . Go by the basic counting pronciple to ease the confusion of P or C

24. Yes 8^6 =256K

25. Ans 4

26. Ans is 42

27. 6 a*a is the formula

28. 76.8

Sol: SP-CP/CP= 0.2 and 80-CP/CP = 0.25 solving both equations we get
SP= 76.8 ANS

29. E.

1) and 2 ) Obviously not sufficient

Combining: We have a st line from 3q and 4Q. This can go from either
2Q ( -ve slope) or 1Q ( + slope). Hence, answer is E.

Edited by Stone on 06 August 2005 at 9:01am

Back to Top

sandilya
VIP~ Posted: 05 August 2005 at 2:09pm | IP Logged
spiderman,

31.If x and n are positive integers (or integers?), and when (n+1)(n-1) is
Joined: 21 December 2004 divided by 24, the quotient is x and the remainder is r. r=?
India 1) 2 is not the factor of n
Posts: 68 2) 3 is not the factor of n
Gender: Male

Answer: C

Can u please explain Q31.

I thought it was E

Back to Top

Posted: 05 August 2005 at 3:10pm | IP Logged


Spiderman
Hello sandilya,
GMAT Tutor

The best way, is to plugin....

Take n =5, not divisible by 2 or 3.

(5-1)(5+1) = 24
24/24 = remainder= 0

Take n=7,
Joined: 02 May 2005
India
(7-1)(7+1) = 48
Posts: 945 48/24, remainder = 0.
Gender: Not Specified
Hence, C

Even in the answer explanation, we take 6k +5, why? because.. 6k+5 is


not divisible by 6. or by eighter 2 or 3.

cheers,

Back to Top

gatgatgam
VIP~ Posted: 05 August 2005 at 4:08pm | IP Logged
45.

rst
Joined: 11 April 2005 uvw
Johnston Atoll xyz
Posts: 477
Gender: Not Specified Each of the letters in the table above represents one of the numbers 1, 2,
or 3, and each of these numbers occurs exactly once in each row and
exactly once in each column. What is the value of r?
1) v + z = 6
2) s + t + u + x = 6

Statment I -- Something is wrong....

Statement II - Possible, since x and u, and s and t, cannot take the same
value and they have to be 1 or 2. r =3

Answer =B

Given Answer: D
-------

Stat (1) v + z =6 is possible only if v=z=3

so we get the pattern

Row 1 rst
Row 2 u3w
Row 3 xy3

Row 2 and Row 3 already have a 3, so third 3 will be in Row 1.

Column 1 Column 2 Column 3

r ; s ; t

u ; 3 ; w

x ; y ; 3

Column 2 and Column 3 already have a '3', so third '3' has to occur in
Column 1.

Combining above, third '3' has to occur in Row 1, Column 1, or (1,1) in


matrix terminology.

Hence Stat (1) is also sufficient.

Thus the given answer (D) is right.

Back to Top

FearFactor
Manager Posted: 05 August 2005 at 5:02pm | IP Logged
42.On the first day, a company sells orange beverage mixed with the same
amount of orange juice and water. On the next day, the company adds the
water twice as much as the amount of orange juice sold on the first day. If
the company sells a glass of the beverage at $0.6 on the first day, how
Joined: 15 June 2005 much is the price on the next day if the company get the same revenue?
United States I choose $0.4
Posts: 51
Gender: Not Specified
Let's say the first day company mixed, X amt of orange juice with X amt
of Water to make 2X amt of orange beverage

Total revenue on the first day, 2X*0.6

Second day, company adds the water twice as much as the amount of
orange juice sold on the first day...so total juice second day

2X + 2X = 4X, Let the price be Y,


4X * Y = 2X * 0.6

Y = 0.3

Now if statement means, twice the amount of Orange beverage sold then
total

= 2X + 4X = 6X

so 6X*Y = 2X*0.6, Y = 0.2

I don't get 0.4 at all...unless i'm misunderstanding the Qn.

Back to Top

Posted: 05 August 2005 at 10:38pm | IP Logged


gp1974
Yes, I do get the answer for Question 42 as 0.3. The given answer might
700 Club
be wrong.

Joined: 17 January 2005


United States
Posts: 413
Gender: Not Specified

Back to Top

Posted: 06 August 2005 at 7:35am | IP Logged


Spiderman
42. Its orange juice that is added twice and not the beverage..
GMAT Tutor

so, it 3x * y = 2x * .6

y = 0.4

cheers,

Joined: 02 May 2005


India
Posts: 945
Gender: Not Specified

Back to Top

Posted: 06 August 2005 at 9:02am | IP Logged


phoenix
44. For (b^2 - 4ac) to be greater than 0, dont we also need to know
VIP~
whether (b^2 > 4ac). Since the choices dont mention b at all, thus the
answer is E.
Other opinions?
Joined: 13 February 2005
India
Posts: 330
Gender: Not Specified

Back to Top

Posted: 10 August 2005 at 11:53am | IP Logged


carnot
hello spiderman,
VIP~

I am not OK with 75, why did you convert to meters instead of just
converting miles per hour into feet per hour or feet per second.

Joined: 01 August 2005 30 miles per Hour = 150 feet per hour
Argentina to cover 44 feet, it takes 44/150 of an hour so it will take 44/150 of 3600
Posts: 39 seconds, which is 1056 seconds
Gender: Male

Back to Top

Posted: 10 August 2005 at 11:57am | IP Logged


carnot
OK spiderman, i found conversion tables on the net and understood that it
VIP~
will take approx 1 sec to cover 44 feet at that speed, but didnt understand
why i didn't get that result by doing what I did

thx anyways
Joined: 01 August 2005
Argentina
Posts: 39
Gender: Male

Back to Top

vcbabu
VIP Posted: 12 August 2005 at 1:42am | IP Logged
80.1/2+1/4+1/8+...+1/512=?
A.0<S<1/2
B.1/2<S<1
C.1<S<3/2
D. 3/2<S<2
E. 2<S<5/2

This is geometric progression.

a= 1/2, r= 1/2 ,l = 1/512


Joined: 11 September 2004
India Sum = (1/2 - 1/512)/(1/2) = 255*2/512 = 255/256 < 1.
Posts: 3078
Gender: Not Specified <STRoNG style="FONT-WEIGHT: bold">(B)

sn = a ( 1- r^n) /( 1-r ) is ths gp summation formula .

how n is find out ,\.

Can so explain .

----------------------------------
pls xplain q 79

tons of thna x

Back to Top

Posted: 12 August 2005 at 4:17am | IP Logged


Spiderman
Hello babu,
GMAT Tutor

Look at the same formula,

the numerator says.. a(1 - r^n)..

what is a*r^n = the last number in the sequence !!

Hence, the numerator becomes, (a-l), where l is the last number in the
sequence.

Joined: 02 May 2005


I hope you have understood this concept.
India
Posts: 945 cheers
Gender: Not Specified

Back to Top

david21
VIP~ Posted: 12 August 2005 at 2:17pm | IP Logged
69.A cuboid is 25 by 10 by 5, except the bottom, all the side faces are
covered with paper. (the bottom maybe 10 by 5?) How much paper will be
used?

Joined: 27 June 2005 Since, we are talking about wrapping the box, we have to calculate
Monaco the surface area of the box except the bottom.
Posts: 21
Gender: Not Specified
10*5 + 2(25*10+25*5) = 425

I think it is 800(250+125=375;375*2=750;750+50=800) ,but


anyway you did a good job thank you

Back to Top

Posted: 15 August 2005 at 6:04am | IP Logged


iyer_srini
Spiderman: Can you explain 99 again. Sorry didnt get the question.
VIP~
Thanks for your time.

Joined: 10 May 2005


United States
Posts: 23
Gender: Not Specified

Back to Top

Posted: 15 August 2005 at 6:11am | IP Logged


saurya_s
700 Club can somebody explain 104. I think it is A

Joined: 19 September 2004


United Kingdom
Posts: 250
Gender: Not Specified

Back to Top

vishal.b21
VIP~ Posted: 15 August 2005 at 9:04am | IP Logged
42 : is 0.4

Joined: 12 July 2005


India
Posts: 511
Gender: Not Specified

Back to Top

Posted: 15 August 2005 at 10:26am | IP Logged


vcbabu
VIP

16.A (n) = A (n-3) +18, and the first several numbers are 102, 108,
114 ... which of the following numbers are in the sequence?
Reference key:
An=96+6n

19.A person travelled from X to Y and arrived at Z. If the average speed in


the whole trip is 15feet/second, what is the average speed during the route
from Y to Z?
Joined: 11 September 2004 1) The average speed from X to Y is 10 feet per second
India 2) It took 20 seconds from Y to Z
Posts: 3078 My answer: E
Gender: Not Specified
CAn so explain these q pls !!!!!!!!!!

43.X is hundredth digit of 0.0X. Y is thousandth digit of 0.00Y. What is the


proximate maximum ratio of the two numbers?
Reference key:
If ask x:y, the answer is 9; if ask for 0.0x/ 0.00y, the answer is 90

44.In xy-plane, Y=ax^2+bx+c,does the graph intersect with X axis?


1). a>0
2). c<0
Answer: C
b^2-4ac>0
My answer is C.

47.Ds m and n are integers, is m^n an integer?


1) n^m is positive
2) n^m is an integer
Answer: E

My answer is E

Back to Top

Posted: 15 August 2005 at 3:54pm | IP Logged


Spiderman Quote: iyer_srini
GMAT Tutor
Spiderman: Can you explain 99 again. Sorry didnt get the question.
Thanks for your time.

The question is not mentioned properly.

I think what it's trying to say is: out of all trees planted WHICH DID NOT
FLOWERED, trees in F which did not flower count as 15% OF THOSE; and
40% of F trees flowered, so what % is the number of trees planted in F
overall?
Joined: 02 May 2005
cheers
India
Posts: 945
Gender: Not Specified

Back to Top

adityakhanna
VIP~ Posted: 18 August 2005 at 9:32am | IP Logged
hi,

can someone pls explain q 18. its answer is mentioned as c, but i think it
Joined: 28 March 2005 shd be e- since even after combining the answer, one doesnt know how
India many of the female students succeed?
Posts: 10
Gender: Male 18.How many of the students applying for making a speech succeed?
1) 2/3 of the male students and 1/3 of the female succeed.
2) 26 male students succeed.
Answer: C

Back to Top

Posted: 18 August 2005 at 12:06pm | IP Logged


Spiderman Quote: adityakhanna
GMAT Tutor

hi,

can someone pls explain q 18. its answer is mentioned as c, but i think it
shd be e- since even after combining the answer, one doesnt know how
many of the female students succeed?

18.How many of the students applying for making a speech succeed?


1) 2/3 of the male students and 1/3 of the female succeed.
2) 26 male students succeed.
Joined: 02 May 2005
India
Posts: 945
Gender: Not Specified
Answer: C

you are right aditya.....

From I, you cannot determine the total number.


From II, you still cannot determine the female students.

From I and II, you still cannot determine the number of females...

Hence, E.

cheers

Back to Top

Posted: 19 August 2005 at 7:31am | IP Logged


Tianlong
131.A guy sent an order of sofas and loveseats, if the total cost was
GMAT Tutor
$3800, how many sofas did he buy?

1) One sofa costs $800 and one loveseat costs $300.


2) There was more than one sofa in the order.
Answer: C

Given answer C is correct.


Stat 1 alone - two options (1,10) and (4,2)
with Stat 2 - only (4,2)

Joined: 16 September 2004 135. Agree with E. Given answer B is wrong. Two variables but we have
United States only one equation with stat 2. Stat 1 is fluff.
Posts: 3843
Gender: Male

Back to Top

Posted: 20 August 2005 at 11:06am | IP Logged


Sudheer Quote:
700 Club
104.If d1, d2, d3, d4 are four different integers, whether is 0.d1d2+0.d3d4
greater than 1?
1) The least number is 4
2) (0.d1d2)(0.d3d4)>0.5
Joined: 22 March 2005
India From I, does not give us any idea.
Posts: 769 From II, yes, we can determine that the sum is greater than 1.
Gender: Male For the product to exceed 0.5, the individual numbers should be atleast
>0.6

Hence, the result is B.

from 1) if least integer is 4, then 0.d1d2+0.d3d4 should be atleast 0.46 +


0.57 which is greater than 1.

So 1) alone is also sufficient.Isn't it?

Back to Top

Posted: 20 August 2005 at 11:36am | IP Logged


Spiderman Quote: Sudheer
GMAT Tutor Quote:
104.If d1, d2, d3, d4 are four different integers, whether is 0.d1d2+0.d3d4
greater than 1?
1) The least number is 4
2) (0.d1d2)(0.d3d4)>0.5

From I, does not give us any idea.


From II, yes, we can determine that the sum is greater than 1.
For the product to exceed 0.5, the individual numbers should be atleast
>0.6

Joined: 02 May 2005 Hence, the result is B.


India
Posts: 945
Gender: Not Specified
from 1) if least integer is 4, then 0.d1d2+0.d3d4 should be atleast 0.46 +
0.57 which is greater than 1.

So 1) alone is also sufficient.Isn't it?

Yes sudheer, you are right..


The answer should be D.

cheers,

Back to Top

adityakhanna
VIP~ Posted: 21 August 2005 at 12:04pm | IP Logged
hi, wanted to know if this explanation for q 31 would suffice? i found the
earlier one a little confiusing.

Joined: 28 March 2005 31.If x and n are positive integers (or integers?), and when (n+1)(n-1) is
India divided by 24, the quotient is x and the remainder is r. r=?
Posts: 10 1) 2 is not the factor of n
Gender: Male 2) 3 is not the factor of n
Answer: C
statement 1: (24x + r) = n^2 - 1 ; thus, as per the 1st statement since 2
is not a factor of n, 2 has to be a factor of n^2- 1; thus, we know
that (24x+r) is a factor of 2; but cant find the value of r

statement 2: similalry; since 3 is not a factor of n, it has to be a facotr of


n^2- 1 (tried different numbers and it worked); thus we again know
(24x+r) is a facotr of 3; but cant find the value of r

when we combine the statements; we know that (24x+r) is a faoctr of both


2&3, thus is a factor of 6; and since the first part (24x) itself is a multiple
of 6; r has to be equal to 0 to satisfy the condition that the full term has to
be divisible by 6; and answer is thus c.

Back to Top

billgates
VIP Posted: 21 August 2005 at 12:53pm | IP Logged
96.The range of the male in a team is 14 inches, and the range of the
female in the team is 12 inches, what is the range of the team?
1) The shortest of the man is three inches more than the tallest woman.
2) The tallest man is 75 inches

range of the team (the tallestest man-the shortest woman)

1)range of the male (Tm-Sm=14);range of woman (Tw-Sw=12)

Sm=Tw+3 ,
Joined: 14 August 2005
Bermuda
Posts: 717 hence insuff ,we don t know SM or TW yet
Gender: Not Specified
2)Tm=75 so 75-Sm=14 ,therefore Sm =61

but we still can not have the range of the team

stat 2 is insuff

When we combine it is suff

can someone explain to me how to find A as the right answer

thank u

Back to Top

archangel88
GMAT Tutor Posted: 21 August 2005 at 1:57pm | IP Logged
For (47), if n is an integer and n<-1, then m^n is a fraction and, therefore,
not an integer.

Example: m=-2, n=-2 => -2^-2 = 1/-2^2 = 1/4

Joined: 14 July 2005


United States
Posts: 3148
Gender: Not Specified

Back to Top

billgates
VIP Posted: 21 August 2005 at 4:38pm | IP Logged
131.A guy sent an order of sofas and loveseats, if the total cost was
$3800, how many sofas did he buy?
1) One sofa costs $800 and one loveseat costs $300.
2) There was more than one sofa in the order.

Not sure of the answer.


From I alone we can determine the answer, becuase without involving both
the terms the sum does not end up to 3800.

My answer : A.
Joined: 14 August 2005
Bermuda
Posts: 717
Given Answer: C
Gender: Not Specified

it can t be A,because there is more than 1 solution for stat 1

for example :800+(300*10) or (800*4)+(300*2)

Back to Top

Posted: 22 August 2005 at 5:23am | IP Logged


iyer_srini
185.Three of the 15 books are defect. If two books are to be selected from
VIP~
the book, what is the probability that both two books are not defect?

Number of ways you can pick 2 books from 15 = 15C2.


Number of ways to pick books without defect = 13C2. Probability =
Joined: 10 May 2005 13C2/15C2.
United States
Posts: 23 >>>>>>>>>>>>Should'nt this be 12C2
Gender: Not Specified

Back to Top

Posted: 22 August 2005 at 7:03am | IP Logged


iyer_srini
217.Bruce bought only $0.32 stamps and $0.23 stamps. How many $0.23
VIP~
stamps did he buy?
1) He bought $0.87 worth of stamps.
2) ...

Joined: 10 May 2005 >>>>>>>>>>>>


United States .87 =2*.032 + 1*.23
Posts: 23
Gender: Not Specified A. should be sufficient

Back to Top

adityakhanna
VIP~ Posted: 22 August 2005 at 10:22pm | IP Logged
hi,

itll be great if someone can just explain this? am not following completely.
Joined: 28 March 2005
India 64.Six colours (red.black.white.orange.pink.yellow) can be used to
Posts: 10 decorate. If one or more can be used, how many ways are possible that
Gender: Male white is used?
Answer: 32
1+C1,5+C2,5+C3,5+C4,5+C5,5=32

Back to Top

vcbabu
VIP Posted: 23 August 2005 at 9:48am | IP Logged
Q 64: .Six colours (red.black.white.orange.pink.yellow) can be used to
decorate. If one or more can be used, how many ways are possible that
white is used?
Answer: 32
1+C1,5+C2,5+C3,5+C4,5+C5,5=32

i think answer is to be 2^n - 1 = 2^6- 1 ( becose of one or more means


at least one ) .
Joined: 11 September 2004
India
Posts: 3078 --------------------------
Gender: Not Specified
185.Three of the 15 books are defect. If two books are to be selected from
the book, what is the probability that both two books are not defect?

Number of ways you can pick 2 books from 15 = 15C2.


Number of ways to pick books without defect = 13C2. Probability =
13C2/15C2.

>>>>>>>>>>>>Should'nt this be 12C2

I too think that it shd be 12 C 2

CAn so confirm thse answers .

____________________________________________________________
_________

Back to Top

adityakhanna
VIP~ Posted: 23 August 2005 at 11:10am | IP Logged
80.1/2+1/4+1/8+...+1/512=?
A.0<S<1/2
B.1/2<S<1
C.1<S<3/2
Joined: 28 March 2005 D. 3/2<S<2
India E. 2<S<5/2
Posts: 10
Gender: Male
This is geometric progression.
a= 1/2, r= 1/2 ,l = 1/512
Sum = (1/2 - 1/512)/(1/2) = 255*2/512 = 255/256 < 1.

hi, just wanted to clarify this formula. there might be a slight error?

solved for the q; Sum = 511/512 <1; even though the answer still remains
as B.

the numerator says.. a(1 - r^n)..= (a- a*r^n) which is not the same as (a-
l)
now, a*r^n is not the same as the last number in the sequence, since the
last sequence is a*r^n-1; that might be the reason for the difference in
answers

Back to Top

Posted: 23 August 2005 at 12:03pm | IP Logged


Spiderman Quote: iyer_srini
GMAT Tutor
217.Bruce bought only $0.32 stamps and $0.23 stamps. How many $0.23
stamps did he buy?
1) He bought $0.87 worth of stamps.
2) ...

>>>>>>>>>>>>
.87 =2*.032 + 1*.23
A. should be sufficient

Joined: 02 May 2005


India Was not sure of B.. so, did not add any OA..
Posts: 945
Gender: Not Specified cheers

Back to Top

Posted: 23 August 2005 at 12:11pm | IP Logged


Spiderman
Hello vcbabu and adityakhanna,
GMAT Tutor

Quote: vcbabu

Q 64: .Six colours (red.black.white.orange.pink.yellow) can be used to


decorate. If one or more can be used, how many ways are possible that
white is used?
Answer: 32
1+C1,5+C2,5+C3,5+C4,5+C5,5=32

i think answer is to be 2^n - 1 = 2^6- 1 ( becose of one or more means


Joined: 02 May 2005
at least one ) .
India
Posts: 945
Gender: Not Specified --------------------------

185.Three of the 15 books are defect. If two books are to be selected from
the book, what is the probability that both two books are not defect?

Number of ways you can pick 2 books from 15 = 15C2.


Number of ways to pick books without defect = 13C2. Probability =
13C2/15C2.

>>>>>>>>>>>>Should'nt this be 12C2

I too think that it shd be 12 C 2

CAn so confirm thse answers .

1. I used logic for the problem.

1 -> with only white


5C1 - > white is already taken and the possibility of a total of 2 colors is
possible if we can chose one more color from 5 colors.
Similarly for the other combinations as well.

the formula holds good !!

2. It should be 13C2. I am not sure why you decided on 12.


If two books are defected, the remaining left are 13. we need to chose
from the remaining 13.

[QUOTE=adityakhanna]

80.1/2+1/4+1/8+...+1/512=?
A.0<S<1/2
B.1/2<S<1
C.1<S<3/2
D. 3/2<S<2
E. 2<S<5/2

This is geometric progression.


a= 1/2, r= 1/2 ,l = 1/512
Sum = (1/2 - 1/512)/(1/2) = 255*2/512 = 255/256 < 1.

hi, just wanted to clarify this formula. there might be a


slight error?

solved for the q; Sum = 511/512 <1; even though the answer
still remains as B.

the numerator says.. a(1 - r^n)..= (a- a*r^n) which is not the
same as (a-l)
now, a*r^n is not the same as the last number in the sequence,
since the last sequence is a*r^n-1; that might be the reason
for the difference in answers
[\QUOTE]

I assumed that the series ends there and hence assumed l = a*r^n

cheers

Edited by Spiderman on 23 August 2005 at 12:14pm

Back to Top

iyer_srini
VIP~ Posted: 23 August 2005 at 12:31pm | IP Logged
185.Three of the 15 books are defect. If two books are to be selected
from the book, what is the probability that both two books are not defect?

Joined: 10 May 2005 >>> 3 books are defective.


United States
Posts: 23 Thanks
Gender: Not Specified

Back to Top

archangel88
GMAT Tutor Posted: 25 August 2005 at 1:38am | IP Logged
For 197, shouldn't the formula go something like this:

Range = [9/5(E) + 32] - [9/5(A) + 32]

Joined: 14 July 2005


United States
Posts: 3148
Gender: Not Specified

Back to Top

Posted: 25 August 2005 at 1:49am | IP Logged


Spiderman Quote: iyer_srini
GMAT Tutor

185.Three of the 15 books are defect. If two books are to be selected


from the book, what is the probability that both two books are not defect?

>>> 3 books are defective.

Thanks

Joined: 02 May 2005


India
Posts: 945
Gender: Not Specified

Sorry, i overlooked the question...

cheers

Back to Top

Posted: 25 August 2005 at 1:53am | IP Logged


Spiderman Quote: archangel88
GMAT Tutor

For 197, shouldn't the formula go something like this:

Range = [9/5(E) + 32] - [9/5(A) + 32]

Yes. you are right...

cheers
Joined: 02 May 2005
India
Posts: 945
Gender: Not Specified

Back to Top

Posted: 25 August 2005 at 6:03am | IP Logged


Tianlong
GMAT Tutor Hey Spidey ,
Don't you think this post is getting unweildy and we should rather restrict
discussions to the smaller threads. Let this master thread be only for
approved solutions and explanations.

Joined: 16 September 2004


United States
Posts: 3843
Gender: Male

Back to Top

Posted: 25 August 2005 at 6:35am | IP Logged


saurya_s
105.n+k=0?
700 Club
1) nk=0
2) n-k=0

From I, we cannot determine the sume.


From II, we still cannot determine.
From I and II, yes we can determine. (n-k)^2 + 4*nk.
Hence, C.

Can someone explain this? Is B not enough? b means n=k. so we get


n=k=5 or n=k=-5. in either case sum is not zero and we get No as the
Joined: 19 September 2004
answer. So should it be B?
United Kingdom
Posts: 250
Gender: Not Specified

Back to Top

Posted: 25 August 2005 at 7:02am | IP Logged


Spiderman
hey all,
GMAT Tutor

Please do not post any questions here.

The respective topics are a new thread in the same forum. The links for the
same are provided in the post.

If there is no forum for those set of questions, please start one, and the
respective link will be updated..

If needed, PM me. I will always respond to PMs


Joined: 02 May 2005
India
cheers,
Posts: 945
Gender: Not Specified

Back to Top

Posted: 26 August 2005 at 2:24am | IP Logged


manugoel007 Quote: saurya_s
VIP~
105.n+k=0?
1) nk=0
2) n-k=0

Joined: 03 June 2005 From I, we cannot determine the sume.


India From II, we still cannot determine.
Posts: 18 From I and II, yes we can determine. (n-k)^2 + 4*nk.
Gender: Not Specified Hence, C.

Can someone explain this? Is B not enough? b means n=k. so we get


n=k=5 or n=k=-5. in either case sum is not zero and we get No as the
answer. So should it be B?

How about n=k=0....i guess this is self explanatory


Back to Top

Posted: 26 August 2005 at 11:58pm | IP Logged


Tianlong
Z should be 1/24.
GMAT Tutor
Answer for this 9/2 or 4.5 times the given amount.

Quote: Spiderman

242. X can transport in 1 hr : 1/16


Y in : 1/12 and Z in:1/18
total work in 1 r = 1/16 + 1/12 + 1/18
In 24 hr = 6/4 + 2/1 + 4/3 = 18 + 24 + 16/12 = 58/12 units

Joined: 16 September 2004


United States
Posts: 3843
Gender: Male

Back to Top

Posted: 28 August 2005 at 6:30am | IP Logged


Spiderman
Thanks Tianlong....
GMAT Tutor

cheers

Joined: 02 May 2005


India
Posts: 945
Gender: Not Specified

Back to Top

kg123
VIP~ Posted: 28 August 2005 at 8:00am | IP Logged
30.If n and m are positive integers and m=n^3/1800, what is the least
value of m?
Answer: 15
1800=(3^2)(2^3)(5^2),
Joined: 20 June 2005 n^3=(3^3)*(2^3)(5^3)=15*(3^2)*(2^3)(5^2), so, m=15
India
Posts: 1067
Gender: Not Specified Understood : 1800=(3^2)(2^3)(5^2), Did not follow the rest of the steps.
Can anyone explain pls. Spiderman, Tianlong : Can u guys chip in.

46.P = sum of positive odd numbers which are < 50


Q = sum of positive even numbers which are < 50

Different ways to calculate P,

METHOD I ---- P = 25/2(1 + 49) = 625 ..PROGRESSION....How to arrive at


this formula?
51.If 37/13=2+1/(13/x), x=?

Simplifyin we get, 26+x/13 = 37/13... Hence, x =11

My answer differs : 37/13 = 3/13/x

=37/13 = 3x/13

=37*13 = 39x

=481/39 = x

Pls correct if i am missing something

60.If three people are to be selected from eight people, who include Bob
and Nancy, how many ways are possible that Bob, not Nancy is selected?

We have 6 ppl + Bob + Nancy. We drop nancy and have Bob.

We need to select 2 ppl from 6. 6C2 = 15 ( How is


this? we need to select 2 ppl from 7 people right,
because we dropped only nancy. So 8-1 is 7)
Answer: 15 ( Also can anyone explain how 6c2 is 15.
Can u detail.)

Back to Top

NC2005
VIP~ Posted: 01 September 2005 at 5:05am | IP Logged
kg123,

30th Question even i am not able to decipher


Joined: 04 July 2005
India 46th Question Spiderman used this formula for Arithmetic
Posts: 68 Progession. & nbsp; Sum = n/2 ( First term + Last term) n is the
Gender: Not Specified number of terms

60th Question u r right that we have dropped Nancy but Bob is always
present so we have 2 places left for which we have 6 people to choose
from

Can anyone pls explain these

30.If n and m are positive integers and m=n^3/1800, what is the least
value of m?
Answer: 15
1800=(3^2)(2^3)(5^2),
n^3=(3^3)*(2^3)(5^3)=15*(3^2)*(2^3)(5^2), so, m=15

31.If x and n are positive integers (or integers?), and when (n+1)(n-1) is
divided by 24, the quotient is x and the remainder is r. r=?
1) 2 is not the factor of n
2) 3 is not the factor of n
Answer: C

68.If x is a two-digit number less than 50 and x=R5, R=? (R is the tens'
digit of x)
1) The thousands' digit of x^2 is 1
2) The hundreds' digit of x^2 is 2
Answer: A

69.A cuboid is 25 by 10 by 5, except the bottom, all the side faces are
covered with paper. (the bottom maybe 10 by 5?) How much paper will be
used?

Back to Top

Posted: 01 September 2005 at 5:40am | IP Logged


Spiderman
Could NOT add anymore in the top post.. so posted here
GMAT Tutor

251.A delivery company charges $20 for the parcel less than 10 pounds. If
the parcel is more than 10 pounds, charge 2 dollars more for each pound
exceed 10. Someone had two parcels delivered, how much did he spend?
1) The cost to one of the parcels is $20
20 The total weight of two parcels is 19.5 pounds.

From I, we cannot tell how much it is less than 10 pounds.. For all the
different weihts below 10, will cost 20.
From !!, we cannot determine the cost for 0.5 pounds...
Joined: 02 May 2005
India
hence, E.
Posts: 945 Answer: E
Gender: Not Specified
252.Price of hamburger increased from $1.85 to $2.05, if the price of
turkey burger has increased in the same percentage, what is the current
cost of turkey burger? The original price of turkey burger is $1.39
£¨number could be wrong£©
Answer: 1.39 * [(2.05-1.85)/1.85] +1.39

253.2 teacher and 3 children queue up. There must be one teacher at the
beginning and one teacher at the back. 3 children are in the middle. How
many ways are possible?

the two teachers at eighter ends, can shuffle themselves in 2 ways.

the remaining children can shuffle in 3! ways.

Hence, 2*3!

Answer: P(3,3)*P(2,2)

254.Xy plane: the name of xy is replaced p, q, a line passed through (0,4)


and (4,,0), what is the equation of such line in terms of p and q?

The basic equations comes down to x/4 + y/4 = 1

Answer: P+Q=4
255.The value of house increased x % from 1975 to 1980. The value
increased y% from 1980 to 1985. Ask how much does the value of the
house increased? The house was 500,000 In 1975 (forgot the exact
number)
1) x + y = a number
2) (1+x/100)(1+y/100) = a number

from I, it does not matter, if it adds up to a number.


From II, its sufficient to determine the increase in value...

My Answer :B

Given Answer: C

256.A seller bought 100 apples at $0.20 each. He sold 75 of them in a pack
of 3, $1 per pack. The rest are sold $0.5 each. What is the average profit
for each apple?
Answer: [75/3 * 1 + (100-75)*0.5 - 0.2*100] /100

257.How many of 7 consecutive integers can be divided evenly by 6?


1) The median is 12.
2) The mean is divisible by 6 (old JJ but modified)

From I, it is clear that, from 7 consecutive numbers only a single multiple


of 6 exists if its a median.
The only way that 2 can exist is when both the ends have a multiple of 6.

From II, The mean turns out to be the 4th element, which is the center
element. Hence, same as I.

Hence, D.

Answer: D
Q199.Of seven consecutive integers, how many are multiple of 6?
1) The median is the multiple of 6
2) The sum of the numbers is the multiple of 6
Answer: D

Same as above.

258.DS: Student major in MIS has to take prerequisites class. 20 of them


took account. Some need to take Computer Science. How many of them
took both?
1) There are 50 students
2) All the student took at least one prerequisite class.

From I, we cannot determine the number of students who attended comp


classes...
From II, still we cannot determine the number since, the number can range
from 1 to 50.

hence, E.

Answer: E

259.{number} represents the least integer that is greater than the


number. If {x/2}= 0, which of the following is the positive value of x?
Choices are -2/3, -3. others are all positive.

I would pick -2/3 ,since it is greater than -3 and for -3 it would be -2.

260.A sequence: 2, 4, 6, ...22. Sequence M contains eight numbers, which


are elements of the former sequence. What is the standard deviation of M?
1) Two sequences have the same average value.
2) M does contain 22
For standard deviation, we need to know the complete sequence.
From I, it cannot be determined.
From II, we still do not know the remaining 7.

From I and II, no possibility.

hence, E.

----E

261.Three machines complete a work in 36 hours at the same constant,


the same rate, and work for the same time. If add one more machine, how
many FEWER hours needed to complete the work?
Answer: 9

Already Discussed above

262.x@y = y/x ¨C x/y, ask which of the following must be true? @


represent a function
I. 1/x @ 1/y = bla bla
II, and III. Total three equations. I chose the second and the third.

Question Not complete

263.If x is divided by 5, the remainder is 3, divided by 7, the remainder is


4. If y is divided by 7, the remainder is 4. x is greater than y. Which of the
following must be the factor of x-y?
15, 28, 35

Pluggin and check

for X, it has to be 18, 53.. and so on..

Y has to be 25, 60..

I have understood so far.. But the Q is not clear...

Given Answer: 35

264.Is x>y?
1) xy^2>0
2) xy^3>0

From I, we do not know the comparision between X and Y.

From II, it is possible that x and Y, both are negative.

from I and II, it is clear that both x and Y are +ve. But still,we do not know
the difference.

Answer: E

265.What is the greatest value of the difference between the two close
prime numbers from 1 to 30?
Answer: 6
29-23=6
266.If a+b=c+d, where a, b, c, and d are integers. How many of them
could be even?
Answer: A:0,2,4
Odd+odd=odd+odd-->0
Even+even=odd+odd-->2
Even+odd=even+odd-->2
Even+even=even+even-->4

267.If the factorial of 12 is divisible by 2^n, what is the greatest value of


n?

12/2 + 12/4 + 12/8 = 6 + 3 + 1 = 10

Given Answer: 10
1+2+1+3+1+2=10(2,4,6,8,10,12)

268.Both the line's intersections with axis-x and axis-y are 4, which
equation represents the line?
Answer: x+y=4

Already solved above

269.To complete a certain work, A and B needs 3 and 6 hours, respectively.


How many hours will it take for them together to complete half of the
work?

Together they take 1/3 + 1/6 = 3/6 = 2 hrs to complete. For Half the work,
they take 1 hr.

Answer: 1 hour

270.Is a certain number the multiple of 36?


1) It is the multiple of 12
2) It is the multiple of 20

From I, we cannot determine. eg = 24


From II, we still cannot determine.. eg =40

From I and II, we still cannot determine, eg = 60

Answer: E

271.Is x>y?
1) x^2>y^2
2) y-y^2<0

Answer: E

Answered above

272.Someone spent M dollars on some newspapers, magazines, and books.


How much did he spend on magazines?
1) The cost on books to the cost on newspapers is ...
2) The cost on magazines to the cost on newspapers and books is ...

From I, we cannot determine the cost of magazine.


From II, we can determine the cost of magazine, as we know the total cost
and and the ratio.

Answer: B
273.A rectangle is L long and W wide, and has diagonal D. What is the area
of the rectangle?
1) L+W=Q
2) D^2=B

From I, we cannot determine the individual components ,. i.e, L or W.


From II, we can only detemine B or D.
From I and II, we can determine the area as we know B, and also L now.

Answer: C

274.|x| = |y|, x/y =1? 1) x>0 2) y<0

From I, we know that x is +ve. We do not know abt Y.


From II, we do not know abt x.
From I and II, we know abt both the quantities...

Answer: C

275.If x is unit digit, y is tenth digit of a certain number, what x and y of


(123,456,789)^2?
I. x = 1, II. Y=2 ,III Y=4
A) I
B) II
C) III
D) I,II
E) I, III
Answer: D
Q101.What is the tens' digit and the units' digit of 123456789^2?
Answer: 21
123456789^2=(123456700+89)^2--->89^2

Back to Top

gmataghu
VIP~ Posted: 04 September 2005 at 2:32am | IP Logged
170.Someone invested a certain amount of money at the 2.5 percent
annual interest rate. At the end of the fifth year, the total amount in the
account is $11250. How much did he invest?
The number maybe inaccurate, pay attention on the thinking
x*(1+2.5%)^5=11250
x+12.5%*x+10*2.5%^2x+...(can be neglected)=11250
Above all: x*(1+2.5%)^5 approximately equal to x+12.5%*x

Here they have not mentioned whether it is simple interest or compound


interest so by default considering it as simple interest the amount will be
Joined: 12 June 2005 10000.
United Kingdom
Posts: 123
Gender: Male Criticism welcomed !!

Back to Top

gmataghu
VIP~ Posted: 04 September 2005 at 4:30am | IP Logged
181.P is an integer, is P a prime number?
1) P+3 is a prime number
2) P^2+3 is a prime number

From I, we can determine that, if P+3 is a prime number, P cannot be a


Prime number.Sine, to be a Prime number, the number should be an odd
number.
From II, P^2 +3 is a prime number, P^2 is an even number, hence, P
cannot be prime

Answer: D

------------------------------------------------------------ -------
Joined: 12 June 2005
United Kingdom My answer C because we cannot answer from condition I that whether its
Posts: 123 prime or not because 2 is also a prime number.
Gender: Male

Combining both the conditions we get the answer as 2.

Because 2+3=5 is prime and 4+3=7 is prime. Only 2 satisfies both


condition.

Answer C

Open to debate.

Rgds,

Gmataghu

Back to Top

vcbabu
VIP Posted: 06 September 2005 at 2:44pm | IP Logged
Q60:

60.If three people are to be selected from eight people, who include Bob
and Nancy, how many ways are possible that Bob, not Nancy is selected?

We have 6 ppl + Bob + Nancy. We drop nancy and have Bob.

We need to select 2 ppl from 6. 6C2 = 15


Joined: 11 September 2004
India YEAH it is confusing
Posts: 3078
Gender: Not Specified
SO pls explain
Back to Top

Posted: 06 September 2005 at 3:25pm | IP Logged


Spiderman Quote: vcbabu
GMAT Tutor

Q60:

60.If three people are to be selected from eight people, who include Bob
and Nancy, how many ways are possible that Bob, not Nancy is selected?
We have 6 ppl + Bob + Nancy. We drop nancy and have Bob.
Joined: 02 May 2005
India
Posts: 945 We need to select 2 ppl from 6. 6C2 = 15
Gender: Not Specified

YEAH it is confusing

SO pls explain

We drop Nancy right.... so we have to pick 3 ppl from 7. Are you with me
up to here..?

Now we select Bob, hence, we have to pick 2 more ppl from 6. Hence, 6C2
= 15.

Cheers,

Back to Top

gmataghu
VIP~ Posted: 10 September 2005 at 6:44pm | IP Logged
227.What is the average of 59 consecutive integers?
1) The sum of the largest number and the least number is 1
2) The sum of all is 1

From condition 1 can be true only for even number of cosecutive integers.
Consider -2 to 3 there are total six integers similary -28 to 29 are 60
integers not 58 integers not 59( Because there is a 0 inbetween) So we
cant get the answer from 1. because it is not possible.

Joined: 12 June 2005 My ans - B


United Kingdom
Posts: 123
Gender: Male

Back to Top

gmataghu
VIP~ Posted: 12 September 2005 at 6:00pm | IP Logged
259.{number} represents the least integer that is greater than the
number. If {x/2}= 0, which of the following is the positive value of x?
Choices are -2/3, -3. others are all positive.

I would pick -2/3 ,since it is greater than -3 and for -3 it would be -2.
------------------------------------------------------------
------------------------------

If I understand this properly, then x should be a number between -1 and 0.


And they have asked what is positive value of x , then it should be between
Joined: 12 June 2005 1 and 0.
United Kingdom
Posts: 123
Gender: Male Correct me if I am wrong.

Back to Top
NChaudhry
VIP~ Posted: 13 September 2005 at 12:24pm | IP Logged
In Ques 104 shouldnt the Ans be A Cause its not given whether the nos are
positive or negative--

Joined: 29 July 2005 if they are negative then if multiplied they may be greater than 0.5 but if
India added thy will be less than 1
Posts: 8
Gender: Not Specified

Back to Top

INDIGO
VIP~ Posted: 17 September 2005 at 5:27pm | IP Logged
SPIDERMAN,

I still don't understand what is happening here... I also read the


discussion link..

how do you get n^3=(3^3)*(2^3)(5^3)=15*(3^2)*(2^3)(5^2),

thanks

Joined: 30 August 2005


30.If n and m are positive integers and m=n^3/1800, what is the least
United States value of m?
Posts: 270
Gender: Not Specified
Answer: 15
1800=(3^2)(2^3)(5^2),
n^3=(3^3)*(2^3)(5^3)=15*(3^2)*(2^3)(5^2), so, m=15

Discussions MJJ30-48

Back to Top

Posted: 21 September 2005 at 8:03pm | IP Logged


rnk_m Quote: INDIGO
700 Club

SPIDERMAN,

I still don't understand what is happening here... I also read the


Joined: 21 May 2005
discussion link..
United States
Posts: 663
Gender: Not Specified how do you get n^3=(3^3)*(2^3)(5^3)=15*(3^2)*(2^3)(5^2),

thanks

30.If n and m are positive integers and m=n^3/1800, what is the least
value of m?
Answer: 15
1800=(3^2)(2^3)(5^2),
n^3=(3^3)*(2^3)(5^3)=15*(3^2)*(2^3)(5^2), so, m=15

Discussions MJJ30-48

explanation
Back to Top

INDIGO
VIP~ Posted: 21 September 2005 at 10:21pm | IP Logged
153.DS: X<5?
1). |x-5|>0
2). x^2 + x <5

From I, we cannot determine if x<5. If X is 4 or 6, we still get an answer of


1.
from II, it solves as x(x+1) <5, in both ways, x is eighter 0 or less than 4.
Hence, B.

Joined: 30 August 2005 FOR STATMENT 2.HOW DO WE KNOW THAT "X IS EITHER 0 OR LESS THAN
United States 4"???
Posts: 270
Gender: Not Specified

Back to Top

Posted: 25 September 2005 at 9:02pm | IP Logged


kgKid Quote: gmataghu
700 Club

181.P is an integer, is P a prime number?


1) P+3 is a prime number
2) P^2+3 is a prime number

From I, we can determine that, if P+3 is a prime number, P cannot be a


Prime number.Sine, to be a Prime number, the number should be an odd
number.
From II, P^2 +3 is a prime number, P^2 is an even number, hence, P
cannot be prime
Joined: 15 August 2005
United States Answer: D
Posts: 87
Gender: Not Specified
------------------------------------------------------------ -------

My answer C because we cannot answer from condition I that whether its


prime or not because 2 is also a prime number.

Combining both the conditions we get the answer as 2.

Because 2+3=5 is prime and 4+3=7 is prime. Only 2 satisfies both


condition.

Answer C

Open to debate.

Rgds,

Gmataghu

How about P to be 4. It satisfies both the conditions. Infact, the answer


should be E.
Back to Top

antmavel
VIP~ Posted: 27 September 2005 at 10:34pm | IP Logged
141.What is the range of the numbers 2, 3, 4, 6, 7, 9, 10, 12, 20, and x?
1) The mean is 9
2) x is the median

Answer :D

If the mean is 9, we can determine the number x.


From II, if x is the median, we can determine x, from the given series.

Answer is correct but explanation is wrong.


Joined: 08 September 2004
China Statement 1 -> agree with you
Posts: 245
Gender: Male
Statement 2 -> we can not determine X, it could be 7,8 or 9. However we
know that it's neither the smallest nor the biggest so we can calculate the
range : 20-2 = 18

Back to Top

gears46
VIP~ Posted: 08 October 2005 at 3:01pm | IP Logged
88.Of a group of people donate for a organization, 1/4 donate less than
$200, 2/3 donate from $200 to $1000, and others donate more than
$1000. If the average value of the donation more than $200 is 360, what is
the average value of the donations more than $1000?
Joined: 02 December 2004 1) The average value of the donations less than $200 is 180
United States 2) The average value of the donations from $200 to $1000 is $540
Posts: 401
Gender: Not Specified
From I alone, we cannot determine the avg, since, we do not have the avg
from 200$ to 1000$.

From II, yes, we can determine.

We have the information that , the avg of donations more than $200 is
360.

Let x, be the number of ppl who donated money.


Then the number of ppl who donated b/w 200 and 1000 is 2/3 x.
and those beyond 1000 is 1 - (1/4+2/3). = 1/12x.

hence, we can calculate, (540*2/3x + A *1/4X) /2/3x + 1/4X = 360

Answer: B

Spiderman,

I believe that the answer would be C. Cause in the formula that you give
you have two variables (A and x). Where you have "A" standing for the
average of the donations less than $200. If you plug in that value (which is
180) into your equation, then you can solve the total number of people
(which is "x").

However, when I did what was stated above, I get a fractional number for
the total number of people. I believe that either the problem has a number
or twop that is incorect, or if those were the intended numbers, the answer
is "E" (cause you can't have a fractional part of a person.

Can you please expand and solve your equation for #88, cause I really
can't see how the answer would be "B".

Thanks,

Gears46

Quote: clueless26

@ Orange&Orange & Kristi

11 & 5 have a set pattern when raised to a particular power... i mean for 5: for every odd power <=
3 the last 3 digits would be 125

for every even power < 2 the last 3 digits would be 625.

for 11..the tens digit would depend on the power raised...

if n = 2,12 etc the tens digit is 2,

n =3,13 tens digit is 3 so on....

not sure if this would help but 76 & 25 are the only 2 digit numbers that when raised to any power
end with the same number.. ie...76^2 ,76^3,76^100 will have 76 as the last 2 digits. Same with 25

26 raised to any power has the last 2 digits as 76

25 raised to any power has the last 3 digits as 625

Very nice tip. Thanks.

One minor mod, I believe clueless meant:

11 & 5 have a set pattern when raised to a particular power... i mean for 5: for every odd power >=
3 the last 3 digits would be 125

for every even power > 2 the last 3 digits would be 625.

82. The surface of a Clock (in a dart


game) is evenly divided to eight parts
numbered with numbers 1 to 8. If we
throw arrows three times, how many
ways are possible that the total score
is 16?
42, 56, 64…

Back to Top

Posted: 12 September 2006 at 6:54pm | IP Logged


anil_bits
Take
VIP~
x=1 y,z can be (7,8) => 2 combinations
x=2 => 3 similar as above 6,8 7,7 8,6
x=3 => 4
x=4 => 5
x=5 => 6
x=6 => 7
x=7 => 8
x=8 => 7
Total 42

Joined: 24 August 2006


India
Posts: 82
Gender: Male

Back to Top

Posted: 12 September 2006 at 9:16pm | IP Logged


eclipse00
Yes.. It is 42.
700 Club

Joined: 05 June 2006


Korea, South
Posts: 282
Gender: Male

Back to Top

Posted: 12 September 2006 at 9:30pm | IP Logged


Serafim
Anil_bits - perfect solution.
VIP~

Joined: 24 March 2006


Ukraine
Posts: 428
Gender: Male

Back to Top
kpinvictus
VIP~ Posted: 12 September 2006 at 9:51pm | IP Logged
agree

__________________
Time is the essence and sky the limit!

Joined: 07 September 2004


Senegal
Posts: 189
Gender: Male

Back to Top

Posted: 12 September 2006 at 11:38pm | IP


VSGMAT Logged
700 Club
When throwing a dart, there is a possibility of missing all
together. I guess we should also consider the case 0 8 8.
According to me, favourable cases are as follows:

Joined: 24 May 2006 8 8 0 ==> 3 combinations


India 8 7 1 ==> 6 combinations
Posts: 109 8 6 2 ==> 6 combinations
Gender: Male 8 5 3 ==> 6 combinations
8 4 4 ==> 3 combinations
7 7 2 ==> 3 combinations
7 6 3 ==> 6 combinations
7 5 4 ==> 6 combinations
6 6 4 ==> 3 combinations
6 5 5 ==> 3 combinations

Total no of favourable combinations : 45 (42 + 3 [for 8 8


0])

Back to Top

Posted: 13 September 2006 at 12:26am | IP


tmddp Logged
VIP
Good explanation VSGMAT. But I think, the 8 8 0 condition
might not be needed. Depends on the options though

__________________
Success is the sole earthly judge of right and wrong

Joined: 09 January 2006


India
Posts: 1651
Gender: Not Specified

Back to Top
anupam384
700 Club Posted: 13 September 2006 at 12:37am | IP
Logged
where i m missing

1 7 8 -->6 cases ( 3! )

2 8 6 -->6 cases

3 8 5 -->6 cases

Joined: 23 August 2006 4 8 4 -->3 cases


India
Posts: 99
Gender: Male total 21 cases??????????????????

__________________
I will not go quietly into the night , I will not vanish without
a fight , I am going to live on , I am going to survive..

Back to Top

Posted: 13 September 2006 at 12:45am | IP


Darshan Logged
VIP
Any short cut method ??
Creation of this possibility table takes time.

Joined: 08 September 2005


India
Posts: 665
Gender: Male

Back to Top

Posted: 13 September 2006 at 1:06am | IP Logged


ashok_nitw
42 is the answer (a throw gets min 1, so 8 8 0 combination
700Club
is not possible)

anupam u r missing some combination ..


to avoid such mistakes take numbers systematically like
below

8 7 1 - 3! --> 6
8 6 2---------> 6
8 5 3----------> 6
8 4 4---3!/2 ---> 3
Joined: 27 July 2006
7 7 2----------->3
7 6 3----------->6
India
Posts: 300 7 5 4------------>6
Gender: Male 6 6 4------------->3
6 5 5-------------->3
any further combinations r duplicate sets.
5* 6 + 4 * 3 = 42.

there is no need to list like this every time just count the
combinations which have all different numbers,which have 2
same and remaining different.. correspondingly use
arrangements formula to get the answer.

Edited by ashok_nitw on 13 September 2006 at 1:30am

__________________
The only true wisdom is in knowing you know nothing

Back to Top

Posted: 13 September 2006 at 1:07am | IP Logged


ashok_nitw
.
700Club
Edited by ashok_nitw on 13 September 2006 at 1:29am

__________________
The only true wisdom is in knowing you know nothing

Joined: 27 July 2006


India
Posts: 300
Gender: Male

Back to Top

Posted: 13 September 2006 at 3:16am | IP Logged


alfa_beta02
I was able to solve the question but it took me around 5-6
VIP~
mins, may be more, to get the answer. Is there any shorter
approach to solve this problem?

Joined: 12 July 2006


United States
Posts: 1188
Gender: Not Specified

Back to Top

Posted: 18 September 2006 at 6:37pm | IP Logged


Orange&Orange Quote: alfa_beta02
VIP
I was able to solve the question but it took me
around 5-6 mins, may be more, to get the answer. Is there
any shorter
approach to solve this problem?

I have the same question as alfa did - does anyone know


any shortcuts
for such questions?

__________________
Joined: 07 September 2006
O&O
Antarctica
Posts: 274
Gender: Not Specified

Back to Top

Posted: 25 September 2006 at 7:15pm | IP Logged


hector81
i dont know if it'll work eveytime.. worked in this ques..
700 Club
since the sum is 16.. the possible combination of three nos
can be done in three ways or 6 ways.. for eg.. 1, 7,8 can be
form a sum in six ways.. also 2,7,7 can also form the sum
in three ways.. that means.. all possible combination can be
Joined: 25 August 2006 done in atleast three ways..
India noting the options there was only one nos which was
Posts: 53 divisble by three.. thats 42.. thats how i chose..
Gender: Not Specified
this aproach will be wrong when there r other options which
will be divisible by 3..

not a gauranteed approach.. but will work in cases like this..

wats say..

Edited by hector81 on 25 September 2006 at 7:18pm

Back to Top

Posted: 25 September 2006 at 8:52pm | IP Logged


Jadore
42
VIP

__________________
A diamond is a piece of coal that stuck to the job.

Joined: 03 August 2006


Russia
Posts: 1366
Gender: Not Specified

Back to Top

Posted: 25 September 2006 at 8:58pm | IP Logged


alfa_beta02
@Jadore,
VIP~
Can you share the approach you used to solve the problem?
Am looking for the best approach to solve the problem.

Thanks!
Joined: 12 July 2006
United States
Posts: 1188
Gender: Not Specified

You might also like